0% found this document useful (0 votes)
167 views40 pages

Mock CLAT 26 Questions

Uploaded by

DPS MUN JODHPUR
Copyright
© © All Rights Reserved
We take content rights seriously. If you suspect this is your content, claim it here.
Available Formats
Download as PDF, TXT or read online on Scribd
0% found this document useful (0 votes)
167 views40 pages

Mock CLAT 26 Questions

Uploaded by

DPS MUN JODHPUR
Copyright
© © All Rights Reserved
We take content rights seriously. If you suspect this is your content, claim it here.
Available Formats
Download as PDF, TXT or read online on Scribd
You are on page 1/ 40

LEGALEDGE TEST SERIES

Part of the Most Comprehensive & Consistently Successful Study Material & Test Series Module, spanning
across both Physical and Online Programs in the entire Country. As a result LegalEdge was able to engineer Clean-
Sweep-Landslide figures of a handsome 64 Selections & 65 Selections in Top 100 (including AIR 1, 2 & 3 from Classroom
Contact Programs in 2022), & a whopping 273 selections & 327 selections in Top 500, in CLAT 2021 & CLAT 2022
respectively. With AILET being no different, a total of 34 of our students found their way into NLU, Delhi in 2021 & 35
in 2022. In a nutshell, every second admit in a Top National Law School in 2021 & 2022 came from the LegalEdge
Preparation Ecosystem.

MOCK COMMON LAW ADMISSION TEST 2023


MOCK CLAT 26

Scan this code after the test


Duration : 120 Minutes Candidate Name : _____________
Max. Marks : 150 Batch : _____________
Centre Name : __________ Contact No. : _____________

to punch in your answers


INSTRUCTIONS TO CANDIDATES (Test ID: 2544451)
1. No clarification on the question paper can be sought. Answer the questions as they are.
2. There are 150 multiple choice objective type questions.
3. There is negative marking of 0.25 for every incorrect answer. Each question carries ONE mark. Total marks are
150
4. You have to indicate the correct answer by darkening one of the four responses provided, with a BALL PEN
(BLUE OR BLACK) in the OMR Answer Sheet.
Example: For the question, "Where is the Taj Mahal located?", the correct answer is (b).
The student has to darken the corresponding circle as indicated below:
(a) Kolkata (b) Agra (c) Bhopal (d) Delhi
Right Method Wrong Methods

5. Answering the questions by any method other than the method indicated above shall be considered incorrect and
no marks will be awarded for the same.
6. More than one response to a question shall be counted as wrong.
7. Do not write anything on the OMR Answer Sheet other than the details required and, in the spaces, provided for.
8. You are not required to submit the OMR Answer Sheet and Test Paper after the test.
9. The use of any unfair means by any candidate shall result in the cancellation of his/her candidature.
10. Impersonation is an offence and the student, apart from disqualification, may have to face criminal prosecution.
11. You have to scan the QR code only after completion of offline test.
12. You cannot leave the examination hall without punching your answers on the portal.
SECTION-A: ENGLISH LANGUAGE

Directions (Q.1-Q.30): Read the following passage carefully and answer the questions that follow.

Passage(Q.1-Q.5): The UN Population Division’s (UNPD) projection that India will replace China as the
world’s most populous nation in 2023, four years earlier than expected, can be sobering. But an alarmist view
would be outdated. A variety of metrics — fertility and replacement rates, sex ratio, the proportion of the young
and old in the country, intra-regional disparities, migration trends — enable a far more nuanced understanding
of demographic dynamics today compared to the 1950s when India embarked on its “population control”
programme. The use of such analytical tools has led to significant shifts in demographic studies — the discipline
has outgrown its Malthusian moorings and population growth is regarded as a challenge, not an emergency. At
the same time, however, terms such as “population explosion” remain in popular parlance and are often invoked
— largely, and often selectively by the political class — to convey the sense of impending crisis. Such
pronouncements, then, pave the way for coercive measures to limit families. In the coming days, policymakers
would do well to avoid knee-jerk reactions to the UN agency’s statistics. An informed debate is needed.

According to the UNPD, a sustained total fertility rate — the average number of children born to a woman — of
2.1 is necessary for a country to achieve population stability. The latest National Family Health Survey puts this
figure at 2. In other words, India is on course to achieving population stability if it maintains this rate in the next
few years — a significant achievement for a country with a TFR of 6 when it commenced its population control
programme. Comparisons with China would be misplaced because force was the leitmotif of Beijing’s three-
and-a-half-decade-long one-child policy. Governments in India — except for a brief lapse into forced
sterilisation during the Emergency — have, in contrast, deployed persuasion and education as tools.
Policymakers have acknowledged the need to give women a greater say in fertility-related decisions. Much more
needs to be done on this, of course, in large parts of the country, including in Uttar Pradesh, Bihar and Madhya
Pradesh, whose TFR is higher than the national average and where gender discrimination has deep social roots.
If India’s estimated 700 million women are forced to remain on the sidelines, the country loses out on ideas and
perspectives that are critical for addressing its several social and economic challenges — including those related
to population — as well as harnessing new opportunities. This should be amongst the first concerns in the wake
of the UN report.

A population of more than 1.4 billion will require the unflinching focus of policymakers on areas fundamental
to human well-being — education, nutrition, healthcare, housing, and employment. The youth will have to be
equipped with skills that are indispensable to the knowledge economy. The climate crisis and other ecological
imperatives will mean that the footprints of many activities are kept light. Most importantly, the challenges will
spur debate, discussion, even dissension and require that diverse voices are heard. (_____________)

1. We can infer from the first paragraph that


(a) The projection that India will replace China as the world’s most populous nation in 2023, four years earlier
than expected, has far-reaching implications.
(b) Understanding underlying demographic forces today requires subtlety and distinctiveness compared to the
1950s.
(c) Various metrics — fertility and replacement rates, sex ratio, the proportion of the young and old in the
country, intra-regional disparities, and migration trends have become static in their approach.
(d) The policymakers need to adopt an alarmist’s approach to incorporate the urgency of corrections in public.

2. The author, in the passage


(a) carries forward the earlier theories concerning population.
(b) highlights the theory regarding population
(c) refutes an earlier concept regarding population
(d) presents an innovative concept on population.

Head Office: 127, Zone II, MP Nagar, Bhopal |+91-7676564400| https://www.toprankers.com Page 2 of 40
3. Which of the following acts as an apt conclusion to the passage?
(a) India’s democratic traditions and the strength of its institutions will be needed to navigate the way forward
from here.
(b) India’s traditions and culture must be reinforced to advance without scaring the Indian essence.
(c) India’s role in the world will have to strengthen to have the power to take indigenous decisions without
intrusion.
(d) India’s youth will have to be cognizant of the dire situations to be savvier concerning future population
expansion.

4. What does the author imply that an informed debate is needed?


(a) The debate must be informed.
(b) The debate must carry weight.
(c) The debate must be intelligent.
(d) The debate must be pertinent.

5. Which of the following best reflects the central idea of the passage?
(a) The UN Population Division’s (UNPD) projection that India will replace China as the world’s most populous
nation in 2023 requires an alarming urgency to take matters seriously.
(b) The UN projections on the Indian population underline opportunities and challenges, addressing which will
require long-term vision and imagination.
(c) The UN projections on populations will require the policymakers to focus on areas fundamental to human
well-being — education, nutrition, healthcare, housing, and employment.
(d) Less participation by women in matters concerning social and economic issues-including those related to
population, makes the country lose out on new ideas and perspectives.

Passage (Q.6-Q.10): The process of brand reinvention must begin by taking a step back to decide when exactly
a brand has to be reinvented. The problem most companies and marketers face is that they are much too
comfortable with their success. This is especially true of large companies, or large brands, which have an inertia
inherent to their success.

While it could be true of the marketing function, one of the worst areas is manufacturing. Because large brands
mean large investments in operations. Because of the ramifications of a change, there is inertness at the
management level. Exaggerating this are the consequences of the decision to re-invent a brand.

My own business and brands were re-engineered only during a crisis. Back in 1993, we were faced with
stagnating volumes and a zero-profit situation. Classical marketing theory tells you that once demand peters off,
the marketer should start analyzing what is wrong with the brand. But, to my mind, that is that a brand has gone
past the stage when it should have been re-engineered. As markets and categories expand, their definitions
change dramatically. One of the best examples of this is the toothpaste category. Toothpaste was a cleaning
exercise. As the market has expanded and the definitions have changed, the consumer has seen the introduction
of different brands. Colgate provided basic cleaning. Colgate Plus was about germ protection and dental care.

So, the definition of the category has changed, and if Colgate continues to view it the way it was ten years ago,
to my mind, it will be in a soup. It relies only on the basic cleaning function; it could get marginalized. But,
ultimately, what does the main brand represent? Dental care, which is only cleaning and germ fighting, while
the market has moved on to the next stage of its evolution. Then, Colgate could go in for penetration in the rural
areas. But in today’s urban context, Colgate can’t adopt that strategy because a large part of its target market,
the upper and middle classes, are buying niche products.

A marketer could be defensive, offensive, or proactive in re-inventing brands. A defensive strategy would be:
my competitor has re-engineered because he has identified a niche; therefore, I should do the same. Or, you

Head Office: 127, Zone II, MP Nagar, Bhopal |+91-7676564400| https://www.toprankers.com Page 3 of 40
could be proactive: I will do a construct of what the consumer will need three years later and design products to
fit those needs today. Or you could adopt an offensive stance.

We used a combination of variables for re-inventing Cadbury’s chocolate brands: advertising, sizing, and pricing.
There were pricing changes and iterations which a consumer may not have even noticed. Pre-reengineering, the
35-gm 5 Star retailed for Rs 8, and the 19-gm for Rs 5. We kept the prices the same but increased the size to,
respectively, 40 gms and 20 gms. And the Value-For-Money equation shifted. Essentially, re-inventing brands
is all about strategic marketing and an intuitive anticipation of consumer needs. And therefore, it is basically all
in the mind.

6. The author seems to be


(a) an ardent fan of Cadbury’s chocolates.
(b) totally in support of defiant consumerism.
(c) a supporter of re-engineering brands.
(d) a critic of Cadbury’s system of marketing.

7. The example of toothpaste given in the passage elucidates the basic thought that:
(a) No one can stop the waning of the product with time.
(b) Times change and product definitions change alongside.
(c) Product identifications change without the changing times.
(d) Sometimes re-inventing a brand is counterproductive.

8. The area that most bears the brunt of re-inventing is:


(a) Production. (b) Marketing. (c) Finance. (d) Structuring of prices.

9. One personal quality of the author that can be sensed from the passage is:
(a) Simplicity and charm. (b) Vociferousness.
(c) Humility and logic. (d) Opinionatedness.

10. The passage seems to be an extract from the following, except:


(a) A finance journal.
(b) A marketing journal.
(c) A private lecture in a consortium of consumer durable section of companies.
(d) A case- study on manufacturing.

Passage (Q.11-Q.15): Recently, the avatar of a female beta tester on Meta’s virtual reality platform, Horizon
World, was groped by a stranger, which led Meta to initiate a probe. The day is not far when all of us will be
living virtual lives in a Metaverse space, and such troubling incidents may not be infrequent. The Management
Consultancy firm Gartner reports that 25 per cent of people will spend an hour per day in the Metaverse by 2026
for work, shopping, education, and entertainment, while 30 per cent of organisations will have products and
services ready for the Metaverse. It seems that we are poised for a huge takeover of real life by virtual life. But
is our legal system ready for it?

While defying precise definitions, the metaverse is generally regarded as a network of 3-D virtual worlds where
people can interact, do business, and forge social connections through their virtual “avatars”.

Within the metaverse, you can make friends, rear virtual pets, design virtual fashion items, buy virtual real estate,
attend events, create and sell digital art — and earn money to boot.
However, be it the virtual world or the real world, where humans interact with each other, disputes do arise and
we have to make laws to be ready for this.

Head Office: 127, Zone II, MP Nagar, Bhopal |+91-7676564400| https://www.toprankers.com Page 4 of 40
In some offences, the human body is the primary target. However, some offences do not require the physical
existence of humans. The modesty of a woman can be hurt in the Metaverse, for instance, by a male avatar’s
gestures towards a female avatar. Similarly, offences such as outraging the religious sentiments of any class,
stalking, and criminal intimidation do not require the physical presence of the victim. The primary target in such
offences is the human mind. Such offences can be committed in the Metaverse because the users carry their
minds into this realm. Similarly, civil disputes such as copyright infringement, trademark infringement, and
property issues can arise without requiring the physical existence of the person.

It’s the avatars that cause offences and give rise to disputes in the Metaverse. Is it possible to identify which user
carries civil liability or has actually committed an offence through the avatar? Establishing the identity of the
offender is necessary because the law would take its recourse only against the human body — not the avatar.
But how would an investigation agency trace the actual preparator? This predicament can be resolved only if the
metaspace provider keeps track of every action of all avatars on its space – though it seems quite difficult now
– and enables all users to gather information about the identity of the user who is controlling that avatar. But
then, this would raise privacy-related concerns.

Then comes the issue of jurisdiction over such offences. In civil disputes, the jurisdiction (governed by Section
16-20 of Civil Procedure Code, 1908, in India) lies where the subject property lies or where the defendant resides
or works or where the cause of action arises. Since the Metaverse has no physical location, the options of filing
at the place where the subject property lies or where the cause of action arises are lost.

As a society, we have been (________) with issues arising from the use of social media, the internet and gaming.
Such experiences become more immersive in the Metaverse. Therefore, the issues will need to be debated much
more intensively. As the Metaverse gets closer to imitating the physical world, the damage (psychological,
emotional, and even physical) could become ever more real. The Metaverse is coming faster than we think, and
we need to ensure our laws are ready for it.

11. One can conclude from the passage that the author is
(a) An editor.
(b) Well-versed in the subject of law.
(c) A lawyer.
(d) A Metaverse savvy person.

12. The author in the passage…


(a) Highlights the core issue that our legal system needs to be equipped to handle the behaviour of users on
Metaverse.
(b) Laments that we are not ready for the virtual existence to interact, do business, and forge social connections
through their virtual “avatars”.
(c) Brings into fore the lacunae in the legal knowledge to cope with the Metaverse virtual world’s offences.
(d) Stresses that civil disputes such as copyright infringement, trademark infringement, and property issues can
arise without requiring the person's physical existence.

13. ‘This predicament can be resolved only if the Metaspace provider keeps track of every action of all avatars on
its space – though it seems quite difficult now – and enables all users to gather information about the identity of
the user who is controlling that avatar. But then, this would raise privacy-related concerns.’ Which of the
following literary devices is used by the author in the given context?
(a) Irony (b) Metaphor (c) Paradox (d) Satire

14. What is the virtual ‘avatar’ as used by the author?


(a) One’s physical self. (b) One’s cybernetic self.
(c) One’s conscience self. (d) One’s mind.

Head Office: 127, Zone II, MP Nagar, Bhopal |+91-7676564400| https://www.toprankers.com Page 5 of 40
15. ‘As a society, we have been _________with issues arising from the use of social media, the internet and gaming.’
Fill in the blank with the correct word to make the sentence coherent.
(a) grappling (b) rummaging (c) managing (d) reeling

Passage (Q.16-Q.20): Most waves are generated by the wind as it blows over the ocean's surface, transferring
energy to and displacing the water. But tsunamis are not normal waves, no matter the size. Tsunamis are created
by an entirely different mechanism. When an underwater earthquake, volcanic eruption or landslide displaces a
large amount of water, that energy has to go somewhere - so it generates a series of waves. Unlike wind-driven
waves where the energy is confined to the upper layer of the ocean, the energy in a series of tsunami waves
extends throughout the entire depth of the ocean. Additionally, a lot more water is displaced than in a wind-
driven wave. Earthquakes can easily move huge amounts of water and cause dangerous tsunamis. No matter the
cause of a tsunami, after the water is displaced, waves propagate outward in all directions- similarly to when a
stone is thrown into a serene pond. Because the energy in tsunami waves reaches all the way to the bottom of
the ocean, the depth of the sea floor is the primary factor that determines how fast they move. Calculating the
speed of a tsunami is actually quite simple. You just multiply the average depth of the ocean- 4,000 metres- by
gravity and take the square root. Doing this, you get an average speed of about 700 kmph. This is much faster
than the speed of typical waves, which can range from about 10 to 30 mph (15 to 50 kmph).

Tsunamis are rare compared to ubiquitous wind- driven waves, but they are often much more destructive. The
2004 Indian Ocean tsunami killed 225,000 people. More than 20,000 lost their lives in the 2011 Japan tsunami.
What makes tsunamis so much more destructive than normal waves? In the open ocean, tsunami waves can be
small and may even be undetectable by a boat at the surface. But as the tsunami approaches land, the ocean gets
progressively shallower and all the wave energy that extended thousands of feet to the bottom of the deep ocean
gets compressed. The displaced water needs to go somewhere. The only place to go is up, so the waves get taller
and taller as they approach the shore. When tsunamis get to shore, they often do not crest and break like a typical
ocean wave. Instead, they are more like a large wall of water that can inundate land near the coast. It is as if the
sea level were to suddenly rise by a few feet or more. This can cause flooding and very strong currents that can
easily sweep people, cars, and buildings away.

Luckily, tsunamis are rare and not nearly as much of a surprise as they once were. There is now an extensive
array of bottom pressure sensors, called DART buoys, that can sense a tsunami wave and alert authorities. This
allows government agencies to send warnings prior to the arrival of the tsunami.

16. Which of the following is/are TRUE as per the passage?


I. The speed of a tsunami is calculated by multiplying the depth of the ocean with the weight and taking the
square root.
II. Tsunamis are very common and destructive.
III. During tsunamis, a great amount of water is displaced as compared to that wind-driven waves.
(a) Only I (b) Only I and II (c) Only I and III (d) Only III

17. Which of the following is/are FALSE as per the passage?


I. Around 2,25,000 people lost their lives during the 2011 Japan tsunami.
II. The Indian Ocean tsunami which killed thousands of people happened in 2006.
III. The average depth of the ocean is 4000 metres.
(a) Only I (b) Only I and II (c) Only II and III (d) Only II

18. Which of the following is similar in meaning to the word ubiquitous mentioned in BOLD in the passage?
(a) Omnipresent (b) Omnipotent (c) Circumscribed (d) Circumnavigate

Head Office: 127, Zone II, MP Nagar, Bhopal |+91-7676564400| https://www.toprankers.com Page 6 of 40
19. According to the passage, how do 'DART buoys' work?
I. They are pressure sensors that can sense a tsunami wave.
II. They send an alert when a tsunami wave is sensed.
III. They send warning messages to the citizens.
(a) Only I (b) Only I and II (c) Only I and III (d) Only III

20. According to the passage, what can cause a tsunami?


(a) Winds blow into areas of the ocean where the water is warm.
(b) Volumes of water displacement by landslides.
(c) A large and sudden displacement of the ocean.
(d) The intense oceanic undercurrents.

Passage (Q.21-Q.25):Faced with a threatening overhang of unpaid debts amounting to over $51 billion, the
Rajapaksas thought they had found a magic potion in organic farming and, without giving a thought to the time
and training it would take to switch to nation-wide organic cultivation, virtually stopped Sri Lanka’s massive
imports of chemical fertiliser. This sharply impacted rice cultivation, the staple of the Sri Lankan diet, raising
food prices to dizzying, unprecedented levels. It also sank the output and, therefore, the export earnings from
tea, their principal source of foreign exchange.

Mismanagement of Covid led to such a wide spread of the pandemic that tourism, the single-most-important
source of foreign exchange, came to a virtual standstill. Remittances from Sri Lanka’s large expatriate
community started drying up. For want of foreign exchange, imports of oil and petroleum products also fell far
short of requirements leading to long, frustrating queues at petrol stations and food stores.

The central bank is now left with reserves of no more than $25 million when Sri Lanka requires at least a billion
dollars a month to keep itself afloat. Mahinda at first refused to go to the IMF, probably because he feared
international reprimand for his economic policies that started with massive tax cuts that bled the Sri Lankan
treasury. He thought his Chinese friends would bail him out, but they have been circumspect, not committing
themselves to more than promising to view with sympathy the restructuring of the huge debt owed to them.
Unthinking borrowing from China, symbolised by the white elephant of the Hambantota port in Mahinda’s
constituency, has disproportionately contributed to the present crisis.

India has come to the rescue with some $3.5 billion in emergency aid, but what a drop in the bucket this is may
be seen from the fact that Sri Lanka needs a minimum of a billion dollars a month in international currency to
import limited essential supplies of fuel, milk, medicines and even food items for its population, who have for
months been struggling with acute shortages and long queues. Now, at long last, an approach has been made to
the IMF, but the negotiations may take months. Inflation, meantime, has exceeded 50 per cent and threatens to
rise to 70 per cent.

That is what has brought the masses onto Galle Face and the streets to demonstrate for long months and now to
occupy a number of government buildings, including the Presidential Palace. Had Mahinda stuck to his original
resolve to make panchayat raj the focus of his government, he would have received the feedback from the people
and could have acted before they were obliged to take to the streets. Hubris has been his lot and that of his family
because military victory against the LTTE unhinged his priorities. Instead of elected representatives at the local
level, his family became his sounding board. This is what happens if the leadership lives in an echo chamber of
mutual praise and backslapping. Sri Lanka’s excellent record on human development — by far the best of any
country in our region — is being dismantled because Mahinda gave up his initial prioritisation of people’s power
through democratic institutions of self-government. It is a lesson for all South Asian governments, including
ours, to learn.

Head Office: 127, Zone II, MP Nagar, Bhopal |+91-7676564400| https://www.toprankers.com Page 7 of 40
21. Which of the following reflects the most suitable title for the passage?
(a) The Sri Lankan Mayhem: Attributable to Organic Farming
(b) A Lesson for Sri Lanka in the Future
(c) A Lesson to be Paid for Mutual Praise and Backslapping
(d) The Sri Lanka collapse: A lesson for all South Asian governments

22. Paragraph one seeks to


(a) account for an impulsive decision taken by the government as a panacea for their woes.
(b) give foreboding reasons leading to the collapse of Sri Lanka.
(c) expatiate on the threatening overhang of unpaid debts amounting to over $51 billion.
(d) Highlight Rajapaksa's ill-informed decisions that contributed to the collapse of the Sri-Lankan economy.

23. In the context of the passage, what does the ‘white elephant’ signify?
(a) A mammoth borrowing from China on the pretext of the Hambantota port.
(b) The Hambantota port became a debt trap that was futile and expensive to maintain.
(c) The Hambantota port led to the nemesis of the Sri Lankan economy.
(d) The Hambantota port was a giant project that robbed the government of its wherewithal.

24. ‘Hubris has been his lot and that of his family because military victory against the LTTE unhinged his priorities.’
What does the author imply through the statement?
(a) Because the collapse of Sri Lanka lies on the head of Rajapaksa and his family, he deviated from primacies
because of excessive pride after the victory against the LTTE.
(b) The leadership living in an echo chamber of mutual praise and backslapping after the victory against the
LTTE made it deviate from the unhinged policies.
(c) The Rajapaksa and his family reeling under the victory against the LTTE, prioritised the same strategies on
other policy matters.
(d) The government became unhinged after their victory against the LTTE, leading to the Sri Lankan collapse.

25. What lesson does the author want the South Asian countries to learn from the Sri-Lankan government?
(a) Not getting swayed by the hubris on account of one victory.
(b) Self-serving overpowering actual agenda of human development.
(c) Not getting derailed from prioritising human development.
(d) All of the above.

Passage (Q.26-Q.30): Branding has a sweet and nasty habit of making it into every realm. Politics is incidentally
one of them. In the wonderland of politics, there are people of every hue of opinion, out to establish their
credentials with their potential voters. Every diverse and divergent opinion is relevant, particularly in a vibrant
and living democracy like ours.

The science of branding the politician is a rather contemporary part of political science. Today, politicians who
aspire to make it big invest a huge amount of time, energy and money to get their branding act right. This is done
right at the beginning, just as it is done with equal passion mid-course to make those vital corrections that count.
Political science must therefore incorporate into it a solid chapter on brand management. It is science being used
by politicians today with finesse and panache.

Look behind the curtains. The politician is a brand in the fullness of it all. My definition of a brand is simple. It
is a thought. The politician is a thought, a powerful one that lives in the minds of millions of people. The
complexity is, of course, the fact that the very same brand is a different animal in every mind. How does Lalu
Prasad Yadav occupy the high ground of being a people’s persona? How does he manage to do it right all the
time? Well, nigh almost. How does he maintain his rustic charm intact in a state that celebrates the rustic and the
real? How does he manage to continue to live and thrive in the mindsets of his peoples?

Head Office: 127, Zone II, MP Nagar, Bhopal |+91-7676564400| https://www.toprankers.com Page 8 of 40
As a part of an exercise to assess the value of such dynamic human politician brands, I have done a brand
valuation exercise on a whole host of them. This exercise views the value living and dead brands bring to the
political parties in focus. It is done across the country, across sets of human politician brands that have captured
the imagination of voters and non-voters alike. The idea is to add the dimension of a physical value to every
human brand. The value is a number that translates itself into crores of rupees, which represents metric that can
actually measure the contribution these human brands make to their political parties and fortunes to date. In
many ways, it is giving the devil its justifiable due.

It is heartening to see the kind of value the person in the pole position brings to the Indian National Congress.
The name is one all of us know as the father of the nation: Mahatma Gandhi. To date, he has contributed valuably
to the fortunes of the party and its current image. Through all its ups and downs, Jawaharlal Nehru, Indira and
Rajiv Gandhi (both assassinated), and of course every other Gandhi from the party has contributed to its image.
Add to it a whole host of names that belonged to the freedom movement and nudged ahead the nation. A political
party is thus a function of the collective images of all its leaders. It is equally an amalgam of all the actions,
inactions, acts of commission and omission of each of these leaders. Sadly, while the work of the common party
worker is often forgotten, leaders thrive as brands remembered.

26. What is the main concern of the passage?


(a) The science of branding the leaders.
(b) Political leaders and the science of branding.
(c) Political leaders and their branding by the media.
(d) Politics on brand management.

27. What is the literary device used in the passage?


(a) Pun. (b) Satire. (c) Rhetoric. (d) Hyperbole.

28. Which of the following statement is false?


(a) The science of branding the politician is a rather contemporary part of political science.
(b) A political party is a function of the collective images of all its leaders.
(c) The author’s definition of a brand is the cumulative thoughts of the party.
(d) Politicians as brands have a metric on which to assess their value.

29. Which of the following is the correct suggestion given by the author in the passage?
(a) The study of politics should efficiently include brand management.
(b) Brand management should be an effective part of our day-to-day curriculum.
(c) The politician is a brand in the fullness of it all.
(d) Branding has a sweet and nasty habit of making it into every realm

30. ‘Look behind the curtains.’ What can be understood from the statement?
(a) Look carefully. (b) Look around.
(c) Observe thoroughly. (d) Observe covertly.

Head Office: 127, Zone II, MP Nagar, Bhopal |+91-7676564400| https://www.toprankers.com Page 9 of 40
SECTION-B : CURRENT AFFAIRS, INCLUDING GENERAL KNOWLEDGE

Directions (Q.31–Q.65): Read the information given below and answer the questions based on it.

Passage (Q.31-Q.35): China is planning to build another highway through Aksai Chin, running along the India
border and connecting Xinjiang with Tibet, according to a newly released highway construction plan. The G695
national expressway will be only the second national highway through the disputed Aksai Chin region, where
China controls 38,000 sq km of land claimed by India, since the controversial construction of the G219 highway
in the [x], and is expected to be completed by [1]. The new highway, the Hong Kong-based South China Morning
Post reported on Wednesday, will run even closer to the Line of Actual Control (LAC) than G219 and is likely
to broadly run along the course of G219 from Mazha in Xinjiang in the north, through Aksai Chin, which is
currently administered under the Xinjiang Uygur Autonomous Region, heading south along the borders with
India, Nepal and Bhutan, and down to Lhunze in southeastern Tibet right across the border from Arunachal
Pradesh.

31. Which of the following has been redacted by [1]?


(a) 2025 (b) 2030 (c) 2035 (d) 2040

32. Which of the following statements is true?


(a) China occupied parts of Eastern Ladakh in the 1960s when they wanted to build a road to East Turkestan
from Tibet.
(b) The road construction was noticed first by some Indian military personnels.
(c) The area at that time was patrolled on the possible biannual frequency with long-range patrols (LRP).
(d) Indian General Zorawar Singh under Dogra ruler Maharaj Gulab Singh conquered the areas beyond Western
Ladakh.

33. How many rounds of commander-level talks have India and China had uptil yet?
(a) 10 (b) 12 (c) 16 (d) 18

34. The India-China Commander level talks focus on the disengagement process in -
1. Depsang Plains
2. Patrolling Point 15
3. South of Demchok
Choose the correct code -
(a) Only 1 and 2 (b) Only 2 and 3
(c) Only 1 and 3 (d) 1, 2 and 3

35. Which of the following is true?


(a) China wants to maintain status quo, put the border issue on the back burner and expects India to normalise
the relationship.
(b) India is playing brinkmanship along the LAC and endeavour to restore status quo ante April-May 2021
through diplomatic/military talks.
(c) Both of the above
(d) None of the above

Head Office: 127, Zone II, MP Nagar, Bhopal |+91-7676564400| https://www.toprankers.com Page 10 of 40
Passage (Q.36-Q.40): Former [X] CEO Jack Dorsey has announced his plans to leverage blockchain technology
to develop the Web 5.0 which will put owners in charge of their data.
This means goodbye to multiple passwords and usernames or the time-consuming entry of our personalised
preferences across multiple apps.
Dorsey, who is now the CEO of financial service firm Block, envisions a next-generation internet to be
decentralised and secure – the information will be saved with the user who can use the same data to access
multiple platforms.
However, the Web 5.0 that Dorsey describes stands out because he claims it will be “truly” decentralised – no
governments or intermediaries to control/store a user’s data. As a result, no government or big tech company
can control information exchange on this Internet, so this also potentially means an end to censorship and is far
more secure from outages.
The Block Head, Dorsey’s bitcoin company says on its website: “The web democratized the exchange of
information, but it’s missing a key layer: identity. We struggle to secure personal data with hundreds of accounts
and passwords we can’t remember. On the web today, identity and personal data have become the property of
third parties.”

36. Which of the following will replace [X] in the following passage?
(a) Facebook (b) Twitter (c) LinkedIn (d) Both (b) and (c)

37. Tackling which of the following is the USP of Web3.0?


(a) Fake news (b) Data Privacy (c) Cyber Attacks (d) Malware

38. Which of the following is NOT true regarding the evolution of the internet?
(a) Web 1.0 era of the internet lasted from 1989–2005.
(b) Web 2.0 facilitated much more interactive features, as well as user-generated content.
(c) We are currently experiencing Web 4.0, having phased out Web 3.0
(d) None of the above.

39. Which of the following is NOT true regarding ways in which Distributed Ledger Technology can be classified?
(a) Distributed ledgers can be classified as private, public,
(b) Distributed ledgers can be classified as Permissioned or Permissionless
(c) Distributed ledgers can only be classified as Combination of any two
(d) All of the above

40. Which of the following is a major use case of Web 5.0?


(a) Control of Identity (b) Control of Data
(c) Control of Policy (d) Both (a) and (b)

Passage (Q.41-Q.45): PARAM PORUL, a state-of the art Supercomputer at NIT [1] dedicated to the nation
under National Supercomputing Mission (NSM) - a joint initiative of Ministry of Electronics and Information
Technology (MeitY) and Department of Science and Technology (DST), was inaugurated on May 25, 2022 by
Shri Bhaskar Bhat, Chairperson Board of Governors, [1] in gracious presence of Prof. G. Aghila, Director, NIT
[1]; Shri E Magesh, Director General, C-DAC, Shri Naveen Kumar, NSM- HPC Division, MeitY; Shri S A
Kumar, Advisor, NSM, MeitY, Dr. Hemant Darbari, Mission Director- NSM, Dr Namrata Pathak, DST; Dr.
Nagaboopathy Mohan, DST, Shri Sanjay Wandhekar, Senior Director, C-DAC, along with senior officials from
MeitY, DST, NIT [1] and C-DAC. PARAM PORUL supercomputing facility is established under Phase 2 of
the NSM, where in majority of the components used to build this system have been manufactured and assembled
within the country, along with an indigenous software stack developed by C-DAC, in line with the Make in India
initiative.

Head Office: 127, Zone II, MP Nagar, Bhopal |+91-7676564400| https://www.toprankers.com Page 11 of 40
41. Which of the following has been redacted by [1]?
(a) Rourkela (b) Tiruchirappalli (c) Silchar (d) Warrangal

42. Since which year did supercomputing begin in India?


(a) 1976 (b) 198 (c) 1983 (d) 1986

43. Consider the following table –


PARAM Computers Feature

1. PARAM 9000 i. First Indian supercomputer to enter


the Top500 list of supercomputers in
the world

2. PARAM Padma ii. This supercomputer had


independent nodes, each based on the
Sun Enterprise 250 server

3. PARAM 10000 iii. First demonstrated in 1994

4. PARAM Yuva iv. It was ranked 69 in the Top500 list


of supercomputers in the world when
it was introduced.
(a) 1-iii, 2-i, 3-iv, 4-ii (b) 1-iv, 2-ii, 3-iii, 4-i
(c) 1-ii, 2-i, 3-iii, 4-iv (d) 1-iii, 2-i, 3-ii, 4-iv

44. Which of the following statements is NOT true?


(a) India's supercomputer output has been dropping off the list of the world’s most powerful machines.
(b) India now has only one supercomputer in the top 100 supercomputers and only two in the top 500
supercomputers in the world.
(c) This year, the Ministry of Electronics and Information Technology (MeitY) has installed four
supercomputers at various institutions in the country.
(d) Fifteen supercomputers, with an aggregate compute capacity of 24 petaflops, have been installed in the
country since 2015 under the National Supercomputing Mission (NSM).

45. The PARAM Ganga was recently established at IIT Roorkee under NSM. PARAM Ganga is a __________
supercomputer.
(a) exascale (b) zettascale (c) petascale (d) terascale

Passage (Q.46-Q.50): The Defence Research and Development Organisation (DRDO) crossed an important
milestone on Monday, when it demonstrated an Air Independent Propulsion (AIP) system that will allow Indian
Navy submarines to operate for up to two weeks without having to surface to recharge its batteries. “The (AIP)
plant was operated in endurance mode and maximum power mode as per the user requirements. The system is
being developed by Naval Materials Research Laboratory (NMRL) of DRDO,” announced the Ministry of
Defence (MoD) on Tuesday.
Since NMRL’s is expected to operationally field its indigenous AIP only by 2023-24, this will come too late to
power the six new submarines that the navy is tendering in a Rs 45,000-crore programme called Project 75-I.
Instead, the six Project-75I submarines will be powered by AIP systems that the foreign vendor must offer. The
DRDO’s AIP system, once ready, will from 2024-25 onward, be “retrofitted” into six Scorpene submarines that
are being built in India under Project 75, say naval sources. Legacy, diesel-electric submarines can operate
silently underwater for up to 48 hours, but they must surface after that to run a generator to recharge their
batteries. When they surface, diesel-electric submarines become vulnerable to detection by enemy radar, which
easily detects submarine masts or snorkels protruding out of the water. This vulnerability is reduced by powering

Head Office: 127, Zone II, MP Nagar, Bhopal |+91-7676564400| https://www.toprankers.com Page 12 of 40
a submarine with an AIP system, which allows for underwater operations for up to two weeks. However,
submarines fitted with AIP must still surface every fortnight for battery charging. In that sense, they are not as
difficult to detect as nuclear-powered subs, which can stay submerged indefinitely.

46. Consider following statements about the P-75I project


I In the first phase, two lines of production were to be established — the first, P-75; the second, P-75I. Each
line was to produce six submarines.
II This P-75I project envisages indigenous construction of submarines equipped with the state-of-the-art Air
Independent Propulsion system at an estimated cost of Rs. 43,000 crore.
Which of the above statements is/are true?
(a) Only I (b) Only II (c) Both I and II (d) Neither I nor II

47. Which of the following statements is not correct about Air-Independent Propulsion?
(a) AIP is a technology for conventional non-nuclear submarines.
(b) Submarines are essentially of two types: conventional and nuclear.
(c) The conventional submarines use coal-electric engines, which require them to surface almost daily to get
atmospheric oxygen for fuel combustion.
(d) If fitted with an AIP system, the submarine will need to take in oxygen only once a week.

48. How many nuclear ballistic missiles submarines does India has?
(a) 1 (b) 2 (c) 3 (d) 4

49. How many conventional submarines does India has?


(a) 13 (b) 14 (c) 15 (d) 16

50. Who is the current Chief Naval Officer of India?


(a) Admiral R. Hari Kumar (b) Admiral Karambir Singh
(c) Admiral R. D. Katari (d) Admiral Manoj Mukund Naravane

Passage (Q.51-Q.55): The Centre sought more time from the Supreme Court to submit its written response to
petitions challenging the constitutional validity of section [X] of the Indian Penal Code dealing with the offence
of sedition. In an application filed before the court, the Centre said that while the draft of the affidavit is ready,
it is still awaiting confirmation from the competent authority.
A bench headed by Chief Justice NV Ramana had last week fixed May 5 to hear the petitions. The bench had
asked the Centre to file its reply by the end of the week and said the petitioners can submit their counter to the
Centre’s affidavit after this so that the court can take up the matter on May 5 for final disposal. The bench also
underlined that there will be no adjournments. As there are more petitions on the issue, the petitioners agreed
that Senior Advocate Kapil Sibal will lead the arguments for them. Attorney General K K Venugopal will also
assist the court in the matter. Issuing notice on the petitions in July last year, CJI Ramana while referring to the
alleged misuse of the provision had asked if the “colonial law…is…still needed after 75 years of independence.”
“This dispute about law is concerned, it’s a colonial law. It was meant to suppress the freedom movement. The
same law was used by the British to silence Mahatma Gandhi, Tilak etc. Still, is it necessary after 75 years of
independence?”, the CJI had asked. He added that “the enormous power of this section can be compared to a
carpenter being given a saw to make an item, uses it to cut the entire forest instead of a tree. That’s the effect of
this provision.”
The CJI also said that “our concern is misuse of law and no accountability of executive agencies”. Responding,
the AG said the provision need not be struck down but parameters for its application be laid down so that it
meets its legal purpose. Vombatkre’s plea challenged the constitutional validity of the sedition law on the ground
that it causes “chilling effect” on speech and is an unreasonable restriction on the fundamental right of free
expression. It contended that “a statute criminalising expression based on unconstitutionally vague definitions
of disaffection” towards Government” etc. is an unreasonable restriction on the fundamental right to free

Head Office: 127, Zone II, MP Nagar, Bhopal |+91-7676564400| https://www.toprankers.com Page 13 of 40
expression guaranteed under Article [Y] and causes constitutionally impermissible “chilling effect” on speech”.
Some of the other petitioners include former union minister Arun Shourie and journalists Kishorechandra
Wangkhemcha from Manipur and Kanhaiya Lal Shukla from Chhattisgarh.

51. Which of these will correctly replace [X] from the above passage?
(a) 121A (b) 122A (c) 123A (d) 124A

52. Consider following statements


I Sedition laws were enacted in 18th century England when lawmakers believed that only good opinions of
the government should survive, as bad opinions were detrimental to the government and monarchy.
II The law was originally drafted in 1837 by Thomas Macaulay, the British historian-politician, but was
inexplicably omitted when the Indian Penal Code (IPC) was enacted in 1860.
Which of the above statements is/are true?
(a) Only I (b) Only II (c) Both I and II (d) Neither I nor II

53. Offence of sedition is a


(a) Bailable Offence (b) Non-Bailable Offence
(c) Partially Bailable (d) Partially Non-Bailable

54. Which of the following statements about sedition law is not correct?
(a) It defines sedition as an offence committed when any person by words, either spoken or written, or by signs,
excite disaffection towards the government established by law in India.
(b) It defines sedition as an offence committed when any person who by visible representation, or otherwise,
brings or attempts to bring into hatred or contempt, or excites or attempts disaffection towards the
government established by law in India.
(c) Disaffection does not include disloyalty and all feelings of enmity.
(d) It helps in protecting the elected government from attempts to overthrow the government with violence and
illegal means

55. Which of these will correctly replace [Y] from the above passage?
(a) 18(1)(a) (b) 18(2)(a) (c) 19(1)(a) (d) 20(1)

Passage (Q.56-Q.60): In a big relief for India, the US has decided not to impose sanctions under its stringent
law Countering America's Adversaries Through Sanctions Act (CAATSA) for purchasing S-400 missile systems
from Russia. After New Delhi’s purchase of the S-400 air defence system, there was a threat of US sanctions
but the Indian government had always been confident of bypassing these. This has been made possible as the
US House of Representatives passed a legislative amendment waiving off any punitive sanctions on India under
CAATSA. The law was brought in [1] and provides for punitive actions by the US government against any
country striking defence and intelligence deals with Russia, Iran and North Korea.

56. Which of the following has been redacted by [1]?


(a) 2016 (b) 2017 (c) 2018 (d) 2019

57. After India, which country looks set to acquire the S-400 system?
(a) Latvia (b) Armenia (c) Greece (d) Belarus
58. Which of the following statements is NOT true?
(a) India had signed the $5 billion deal with Russia for purchase of five units of the S-400.
(b) The S-400 can take down multiple targets up to a range of 400 km.
(c) The S-400 has four different kinds of missiles and has beyond visual range capabilities.
(d) Russia has sought more time from India for delivering the rest of the units due to the disruption caused by
the war in Ukraine.
Head Office: 127, Zone II, MP Nagar, Bhopal |+91-7676564400| https://www.toprankers.com Page 14 of 40
59. Which of the following statements regarding VL-SRSAM is NOT correct?
(a) It has been designed and developed jointly by three facilities of the Defence Research and Development
Organisation for deployment of Indian Naval warships.
(b) Barring sea-skimming targets, the missile can neutralise various aerial threats at close ranges.
(c) Two key features of the VL-SRSAM are cruciform wings and thrust vectoring.
(d) None of the above
60. Indonesia is in talks with India to buy BrahMos missiles. Which of the following statements cannot be proper
implications of this move?
(a) Indonesia will become the second ASEAN country to buy the BrahMos missile from India.
(b) India’s Act East policy will get a boost.
(c) However, this deal might run into problems as Russia might be reluctant after Indonesia cancelled their Su-
27 fighter jets import.
(d) The other ASEAN countries shall start showing interest in BrahMos and Akash missiles.

Passage (Q.61-Q.65): India is playing geopolitics confidently and astutely as testified by the I2U2 virtual
summit on July 14. India seems to have judged well the trends in West Asia involving Israel, the Gulf states and
Iran and positioned itself well in the area. For some years, India and Israel have been drawing closer, to the point
that Israel has become a major defence partner of India. This has not stood in the way of forging close ties with
the Gulf states. Indeed, the transformation of our ties with the UAE and Saudi Arabia has been a notable success
of Indian foreign policy. With the other three countries being military allies, India had sensitivities about being
seen as part of a broader military alliance. The Quad, therefore, evolved gradually from official to political level
(foreign ministers) and then to summit level. India is today far more committed to the Quad, calling it a force
for the public good.
61. Which of the following developments seems to have made I2U2 possible?
(a) The takeover of Afghanistan by the Taliban (b) The Abraham Accords
(c) The developments in Sri Lanka (d) All of the above

62. Which of the following statements is NOT true?


(a) Six areas of cooperation have been identified by the I2U2.
(b) The I2U2 Group will advance a hybrid renewable energy project in Gujarat consisting of 300 megawatts
(MW) of wind and solar capacity.
(c) For UAE, the I2U2 is a new concept as it has never been part of any such regional grouping in the past.
(d) None of the above
63. The development of I2U2 shows that -
(a) India has exposed its wariness over the US’s role in West Asia.
(b) Washington is comfortable with India playing a proactive role in geopolitics.
(c) India has started viewing West Asia through the lens of the Pakistan problem.
(d) India’s place at the diplomatic table is pretty constant.

64. Who out of the following had referred to I2U2 as ‘West Asian Quad’?
(a) Ahmed Albanna (b) Kenneth Juster (c) Naor Gilon (d) Taranjit Singh Sandhu

65. Which of the following statements is NOT true?


(a) The recent Quad leaders’ summit took place in Tokyo.
(b) In the recent summit the Indo-Pacific Partnership for Maritime Domain Awareness was launched.
(c) The USA formed a new Quad Group with Pakistan, Afghanistan and Tajikistan.
(d) The US-Pakistan Quad focuses on enhancing regional connectivity with a key focus on Afghanistan.

Head Office: 127, Zone II, MP Nagar, Bhopal |+91-7676564400| https://www.toprankers.com Page 15 of 40
SECTION - C: LEGAL REASONING

Directions (Q.66-Q.105): Read the comprehension and answer the questions:


Passage (Q.66-Q.71): The Indian Penal Code, 1860 is the substantive law governing the criminal activities in
India. Chapter XXII of the Code, consisting of Section 503-Section 510 provides for offences which fall in the
nature of criminal intimidation, intentional insult and annoyance. The law has an objective of punishing offenders
for all kinds of criminal offences but in contemporary times, the law does not entirely mirror the changes that
have taken place in society. Although the provisions are relevant and binding, they fail to address the
technological advances in today’s world. The law is archaic but provides a base for new developments. Further,
the intention should be to cause alarm to that person; or to make them perform any act which they are not legally
bound to do; or to omit any act which they are legally entitled to perform. If they are forced to do all of these
acts as a means to avoid execution of such threat, this amounts to criminal intimidation. The explanation of this
section states that a threat to injure the reputation of a deceased person in whom the person threatened is
interested is also covered under this section. The Supreme Court in the case of Vikram Johar vs. State of Uttar
Pradesh (2019) has observed that the mere act of abusing a person in a filthy language does not satisfy the
essential ingredients of the offence of criminal intimidation. The complaint was that the accused came with a
revolver to the complainant’s house and abused him in a filthy language. They also attempted to assault him but
when the neighbours arrived, they fled from the spot. The Bench held that the above allegations prima facie do
not constitute the offence of criminal intimidation. In Keshav Baliram Naik vs. State of Maharashtra (1995), it
was alleged that the accused touched the hand of the prosecutrix, a blind girl, when she was asleep and further
proceeded to remove her quilt and insert his hand inside her dress. He threatened to kill her if she disclosed his
identity. The Court held this to be a case of criminal intimidation.
66. Ajit and Anamika were husband and wife and they worked under the same law firm as managing partners and
managing directors. One day they had some heated arguments and in fit of rage, Ajit left the home. After few
hours, he returned and used filthy languages on his wife and ensured that she will not be working in the law firm
as the managing partner after tomorrow as she is going to be portrayed as a con for the company and she would
not be able to work within her fraternity any more. As a result she died of anxiety and heart attack. A complaint
was filed a complaint against Ajit for criminal intimidation. Which of the following statements is false?
(a) Ajit is not liable for criminal intimidation as using filthy language is not criminal intimidation.
(b) Ajit has committed criminal intimidation because using filthy language against his wife along with
portraying her as a con is criminal intimidation.
(c) A complaint of criminal intimidation can be filed because he threatened his wife to portray her as a con.
(d) Criminal intimidation cannot come into picture as filthy language is not a pre requisite for criminal
intimidation.
67. Paramjit and dev were brothers. Param was elder. They had a common friend named Biswa. One day, Dev and
Biswa engaged themselves in a scuffle in the streets of the city which tarnished the image of Dev as he was a
successful business and some respect in the society. When Param came to know the same, he directed his brother
to cause grievous hurt to Biswa which was a punishable offence in itself, to teach him a lesson as Param
somewhere didn't like Biswa, which Dev refused to do. Param put a revolver on Dev’s head and abused him and
then directed the same. Dev agreed and caused grievous hurt to Biswa. Which of the following sentences can be
considered to be true?
(a) Had Param threatened Dev to kill, Param would have been liable for criminal intimidation.
(b) However, Param had directed Dev to cause grievous hurt Biswa, he is not liable for criminal intimidation as
he had merely abused Dev and put a revolver on his head and merely abusing someone is not criminal
intimidation.
(c) Dev had caused grievous hurt to Biswa, so Param is liable for criminal intimidation as Dev was under the
direction of Param and he was abused by him.
(d) Mere putting revolver and abusing to direct someone is not criminal intimidation.

Head Office: 127, Zone II, MP Nagar, Bhopal |+91-7676564400| https://www.toprankers.com Page 16 of 40
68. “The Supreme Court in the case of Vikram Johar vs. State of Uttar Pradesh (2019) has observed that the mere
act of abusing a person in a filthy language does not satisfy the essential ingredients of the offence of criminal
intimidation”. Construing to the given lines, what can be best inferred from the same?
(a) Putting revolver on head of a person and forcing him to do an act illegal is not criminal intimidation.
(b) Using abusive language leading someone to do some act that he is not legally bound to do can’t be criminal
intimidation.
(c) Possession of revolver in front of a person can amount to criminal intimidation.
(d) Using filthy language against someone and not merely carrying a weapon in front of him leading him to do
an omission may not be considered as an intimidation.

69. Udbhav Tackrey was the son of a renowned politician named Kal Tackrey who had an image of a savior and a
warrior, adamant to work for the welfare of the people of his state. when Kal Takrey died, Udbhav came to his
throne and became the minister. However, certain allegations were labeled on Kal Tackrey that he was involved
in the lynching of a news anchor, however, he came out clean chit. Pranab gosami was another news anchor who
had personal feud with Udbhav. One day, on a heated argument between Udbhav and Pranab in a debate show,
Pranab told that he will reveal the audio recording of Kal Tackrey in which he was heard threatening the deceased
news anchor to beat him to death, and ensured Udbhav that the same will be sufficient to diminish the Godly
image of Kal Tackrey. Udbhav filed a complaint of criminal intimidation against Pranab. Construing to the facts
select the most appropriate option.
(a) If it is proved that the words of Pranab led Udbhav do some act that he is not legally bound to do, then it can
come under criminal intimidation.
(b) Pranab can take the defense of Truth and thus, there is no criminal intimidation.
(c) As Kal is dead, there is no point to release the recording as it would not affect Udbhav and thus it is not a
criminal intimidation.
(d) It may be a criminal intimidation however, it didn't lead Udbhav do such an act which he is not legally
entitled to do.

70. Helen was a minor of 16 years. She was alone and was taking a nap in her home when Jhony, who was a dreaded
criminal, infamous for robbery in the locality, silently broke into her home. He tried to touch her inappropriately
while she was asleep and tried to remove her clothes. Meanwhile she was awakened and was taken aback by his
presence in her home. She got extremely scared and after he left, she couldn’t tell the incident to her family
members as she was scared that upon the revelation of his identity, it would create certain problems for her.
However, after few days, she filed a case of criminal intimidation against Jhony. What is the best statement that
can be chosen?
(a) Helen can surely file a criminal intimidation case against Jhony as he intimidated her and after he left, Helen
was scared as she could not speak of him to her family members about the incident.
(b) Helen cannot file a criminal intimidation case.
(c) Jhony is liable for criminal intimidation as he was an infamous robber and his presence can intimidate a
person if he is trying to touch the victim inappropriately and remove her clothes.
(d) Criminal intimidation case can be filed as the girl was taking a nap when Jhony entered the home and there
was a mere chance to intimidate the girl while forcefully entering the home.

Head Office: 127, Zone II, MP Nagar, Bhopal |+91-7676564400| https://www.toprankers.com Page 17 of 40
71. Karan was a shopkeeper. Jamal and Atiq were his frequent customers. One day, they were discussing upon some
political issue on which Karan dissented with Jamal and Atiq. They took the arguments of Karan otherwise. One
day, Jamal and Atiq came to Karan’s shop to buy the ration and under the cover of buying the ration, Atiq
signaled Jamal to attack Karan with a knife which he agreed and after that they tried to assault Karan. Karan
shouted out loud when fellow shopkeepers gathered seeing which both fled away. However, they did not use any
filthy language against Karan, but Karan saw that apart from knife, Jamal was carrying a pistol which intimidated
Karan. He filed a case of Criminal intimidation against Jamal. Choose out the option that can be the best among
the following.
(a) Atiq and Jamal, both are liable for Criminal intimidation as they both attacked the shopkeeper.
(b) Had Atiq used filthy language against the shopkeeper, he would be liable for the criminal intimidation, but
in the present scenario, Atiq was not liable for criminal intimidation but Jamal may be held liable.
(c) As Jamal had attacked Karan with the knife, he may be liable along with Atiq as he was carrying the gun by
which Karan got intimidated.
(d) Jamal assaulted Karan with knife but on the signal of Atiq. But Atiq is not liable for intimidation or Jamal
as he did not intimidate Karan with his pistol.
Passage (Q .72- Q.76): Have you heard of the Fraud Constitution? No, many people may not have heard it
because everyone would have heard it as the doctrine of Colorable Legislation used by the courts of India. The
doctrine of Colorable Legislation is based upon the Latin maxim “ uando aliquid prohibetur ex directo,
prohibetur et per obliquum” that says When something is prohibited directly, it is prohibited indirectly. The
doctrine is used in the cases to determine questions of competency to enact a law when a legislature overtakes
its conferred power and legislates upon something indirectly, which it cannot do in a direct manner. In simple
words, the meaning of the Doctrine of Colorable Legislation that ‘if you could not do anything directly, also you
cannot do indirectly.’ Under the colour or guise of the power given for the other purposes, the legislature cannot
demand to achieve that another purpose which it is otherwise not competent to the legislature in pursuance of
the Indian Constitution. Changing its colour, form, or language does not make the wrong thing right, because
wrong is always wrong. Therefore, under the Indian Constitution, the powers of the Center and the State are
divided into three lists under Schedule Seven of the constitution: first the Union list, second the state list and the
last Concurrent list. Thus, both will follow their power to make laws according to their list. So that neither of the
two makes their law on the subject of each other’s list and there is no guise of differences between the two. If
either of the two shall make a law using its power beyond competence for its purpose, it shall be considered by
the court that the law is Colorable Legislation. Also, called a Fraud Constitution. Parliament has exclusive power
to make laws with respect to any of the matters in List I, and State legislatures in Lit II. Parliament and State
Legislatures have both powers to make laws with respect matters in List III which is also known as concurrent
list.
72. Which of the following statements provides honest information about the passage?
(a) Sometimes the legislature makes such law, which appears to be within its competence but its effect and
essence lie beyond its boundary. Then the law would be declared as void.
(b) The different colour is given to the law but, it cannot prevent it from being declared an illegal law.
(c) In this doctrine of colorable legislation, the focus is only on whether the legislature can legislate or not.
(d) Colorable legislation is used by the courts to settle a dispute over the jurisdiction..
73. The Mines and Minerals Act of 1957 gave the central government the authority to raise royalty rates. In 2020,
several coal-producing states independently imposed and received coal development cess, stating that the states
needed revenue out from cess because coal mines were located into their territory. Which of the following
statement is correct in respect to the author’s argument presented in the passage?
(a) The act of state is not valid and will not be considered violation of the doctrine of colorable legislation.
(b) Any act done using its power beyond competence for its purpose; it shall be considered by the court that the
law is Colourable Legislation.
(c) The state is said to be encroaching on the powers of central government. Hence a fraud constitution.
(d) The legislature cannot demand to achieve the purpose which it is otherwise not competent to the legislature
in pursuance of the Indian Constitution.

Head Office: 127, Zone II, MP Nagar, Bhopal |+91-7676564400| https://www.toprankers.com Page 18 of 40
74. The Maharashtra government attempted to reassess certain theatre owners for evading entertainment tax. The
Madras High Court ruled that the Maharashtra Entertainment Tax Act 1939 did not allow for a reassessment.
What logical conclusion can be drawn from the given illustration?
(a) It will be an example of fraud constitution.
(b) The state of Maharashtra attempted to enact law which is ultra vires.
(c) The state government of Maharashtra will be charged for violation of colorable legislation.
(d) The state government of Maharashtra will be not charged for violation of colorable legislation.

75. Legislative assembly of state Z passed a law called Z Land Tax Act, 1955. The Supreme Court declared the Z
Land Tax Act, 1955 as invalid. The court said that the Act violated Articles 14 and 19(1) (f) and its provisions
were confiscatory in nature. Is the ruling valid?
(a) Yes, because it violates the fundamental rights enshrined in India's constitution.
(b) No, because neither the state legislature nor the parliament has the authority to enact laws ancillary to
fundamental rights.
(c) No, because the state lacks the authority to enact legislation outside of its purview.
(d) Yes, it will be a case of constitution fraud.

76. The state of X enacted legislation. In the interest of the general public of the state of X. The proposed Bill intends
to incentivize people to have one child by offering promotions, tax breaks, and job security in the public sector.
The court that overturned State Z's legislation because it violated the rules of colorable legislation. The State
argued that the operation was not intended to encroach on the subject matter of List I, but rather was
supplementary to the subject matter of List II. Decide?
(a) Merely ancillary matters were encroached which makes the Act valid, as its object and purpose was lawful.
(b) The list II is exclusive to the State; hence they can make laws regarding it, regardless of the encroachment.
(c) The Act is invalid as the legislative intent of the Act does not account for, it’s effect was encroachment of
List I and hence, its unlawful.
(d) The act is valid as the legislature merely enacted law ancillary to the subject of list II.

Passage (Q.77-Q.81): Censorship is proclaimed to be a tool that is placed to keep a fair check on what comes
into the public domain and how this information or data fulfills certain commonly acceptable standards to
maintain harmony, peace and social order. In Sahara vs. SEBI, popularly known as the “Media Guidelines Case”,
the Supreme Court carved out a specific exception to the rule against prior restraint. In SEBI, the Court was
concerned about the issue of media trials causing prejudice in sub judice matters. In that context, the Court held
that it had inherent powers under the Constitution to “prohibit temporarily, statements being made in the media
which would prejudice or obstruct or interfere with the administration of justice in a given case pending in the
Supreme Court or the High Court or even in the subordinate courts.” Drawing this power under Article 129 of
the Constitution, which authorised the Supreme Court to punish for contempt of itself, the Court held that the
power to punish included the power to prevent as well. On this basis, the Court held that it could pass
“postponement orders” (i.e., temporary injuncting the media from reporting on a particular event) in order to
ensure the proper administration of justice, a fair trial, and the protection of the rights of the accused under
Article 21. The Court warned that:
“Given that the postponement orders curtail the freedom of expression of third parties, such orders have to be
passed only in cases in which there is real and substantial risk of prejudice to fairness of the trial or to the proper
administration of justice which in the words of Justice Cardozo is “the end and purpose of all laws”. However,
such orders of postponement should be ordered for a limited duration and without disturbing the content of the
publication. They should be passed only when necessary to prevent real and substantial risk to the fairness of the
trial (court proceedings), if reasonable alternative methods or measures such as change of venue or postponement
of trial will not prevent the said risk and when the salutary effects of such orders outweigh the deleterious effects
to the free expression of those affected by the prior restraint. The order of postponement will only be appropriate
in cases where the balancing test otherwise favours non-publication for a limited period.”

Head Office: 127, Zone II, MP Nagar, Bhopal |+91-7676564400| https://www.toprankers.com Page 19 of 40
77. A 49-second trailer of an upcoming show on Tuntun TV, which had a deep communal overtone, caused a massive
uproar on social media and led to the filing of numerous complaints with the Ministry of Information and
Broadcasting. On the basis of these complaints, a notice had been issued to the channel by the Ministry for
clarifying details over the content of the show with regards to the programme code of the Cable Television
Network Rules, 1994. Meanwhile, the Supreme Court refused to stay the broadcast of the show. What may be
inferred from the following facts and the court's ruling?
(a) That the court has applied the balancing test theory and it otherwise favors publication of the show.
(b) That the court has applied the balancing test theory and it otherwise favors non publication of the show.
(c) That the court has applied the balancing test theory and it otherwise does not favors publication of the show.
(d) That the court has not applied the balancing test theory and it otherwise not favors publication of the show.

78. Sujata Roy, one of those mentioned in the documentary, requested a stay of the film's publication, a court in Bihar
granted the request after determining that Roy met the requirements for a prima facie case, a balance of
convenience, and irreparable loss. The Information and Broadcasting Ministry had opposed Mayank Choksi's
writ petition in the Delhi High Court just two days before to this stay on the grounds that "freedom of speech
and expression is vital and should be given full play." Mayank Choksi is also featured in the
documentary. Choose the correct statement.
(a) The order of both courts is legitimate because the balance test does not favour a postponement order.
(b) In situations where the balancing test would ordinarily favour non-publication for a brief time, the order of
postponement is appropriate.
(c) The petitioner's reasoning is persuasive; hence the Delhi court's ruling is invalid.
(d) The Delhi High Court's order is invalid since it is not based on the notion of the balancing test.

79. Without even giving the book's author or publisher a chance to be heard, an order was made. The Judge claimed
that after skimming the book and looking through its controversial passages, he discovered that they were
defamatory. On this basis, the sale of the book was baned.. Choose a correct statement.
(a) The court orders are invalid because it is unlikely to censor a work for an indefinite amount of time.
(b) The court's order is lawful because it was determined that the book's content was defamatory in nature.
(c) The court's ruling is invalid because, even though the alleged content was apparently deemed to be
defamatory, it cannot be banned indefinitely.
(d) The court's order is invalid since it was issued in violation of the natural justice principles.

80. The makers of a film approached the Bombay High Court to appeal against the decisions of the Central Board
of Film Certification (CBFC) and the Film Certification Appellate Tribunal (FCAT). One of the scenes which
were asked to be deleted by the Board was that which involved a father who, upon the death of his eight-year-
old son in Kashmir, makes a statement against India and the Government. The scene was asked to be deleted on
the ground that the comment would be considered as sedition under Section 124-A IPC. The issue of whether or
not to outlaw films that encourage seditious behavior in public also became a hot topic in the country on various
news channels. However, not a single news channel made any mention of the film's script in their coverage.
What will be correct ruling in the present case?
(a) The court will nonetheless put a restraining order on all the news channels as the said discussion on news
channels might affect the adjudication of the case at present.
(b) Since no news station mentioned the movie's script in their broadcast, the court cannot impose any
restrictions on media outlets that discuss topics related to the current case.
(c) The court is unable to impose any restrictions on media outlets since such orders may only be made where
there is a genuine threat to the integrity of the legal process or the administration of justice.
(d) Even if the script is not mentioned, the court may impose a restraining order on all news channels if they
address a topic related to the current case.

Head Office: 127, Zone II, MP Nagar, Bhopal |+91-7676564400| https://www.toprankers.com Page 20 of 40
81. Tolly LLB , a film based on a true incident was surrounded by controversy since the beginning of its release. A
public interest lawsuit (PIL) had been submitted to the Delhi High Court, asking for the issuance of a writ of
mandamus ordering the CBFC to revoke the certification it had given the film. The film was subjected to a never-
ending bombardment of cases raising concerns about the portrayal of the legal profession and the judiciary in
the film. Decide whether the court will/ will not pass an injunctive order?
(a) The court will pass an injunctive order until the case has been concluded and there is doubt that such
restriction is necessary further.
(b) The CBFC has already issued a clearing certificate; hence the court will not issue an injunctive restriction
order.
(c) The CBFC has granted the film certificate; hence the court will not get involved or consider such frivolous
PILs.
(d) The court may issue an order delaying the screening of the aforementioned movie until a final decision has
been made.

Passage (Q.82- Q.86): Section 84 of IPC deals with the “act of a person of unsound mind. “Nothing is an offence
which is done by a person who, at the time of doing it, by reason of unsoundness of mind, is incapable of knowing
the nature of the act, or that he is doing what is either wrong or contrary to law.”
On analysis of the Section 84 IPC, the following essential ingredients can be listed. For the sake of easy
understanding, the Section 84 IPC can be divided into two broad categories of, major criteria (medical
requirement of mental illness) and minor criteria (loss of reasoning requirement). Major criteria (mental illness
requirement) mean the person must be suffering from mental illness during the commission of act. Minor criteria
(loss of reasoning requirement) mean the person is:
a. Incapable of knowing the nature of the act or
b. Incapable of knowing his act is wrong or
c. Incapable of knowing it is contrary to law.
Both major (mental illness) and minor (loss of reasoning) criteria constitute legal insanity.
Section 84 IPC, clearly embodies a fundamental maxim of criminal jurisprudence that is, (a) “Actus nonfacit
reum nisi mens sit rea” (an act does not constitute guilt unless done with a guilty intention) and (b) “Furiosi nulla
voluntas est” (a person with mental illness has no free will). This means that an act does not constitute a crime
unless it is done with a guilty intention called “mens rea. Hence, Section 84 IPC fastens no culpability on persons
with mental illness because they can have no rational thinking or the necessary guilty intent.

82. Which of the following statements provides best information about the passage?
(a) Any person, who is suffering from any kind of mental illness is called “legal insanity,” however “medical
insanity” means, person suffering from mental illness should also has a loss of reasoning power.
(b) The term legal insanity refers to the “mental state” of a person at the time of committing crime and nothing
else.
(c) Legal insanity means, at the time of the commission of the act, the person should be suffering from mental
illness and also have a reasoning power.
(d) Mere absence of motive for a crime and howsoever atrocious the crime may be, in the absence of plea and
proof of legal insanity, cannot bring the case within the ambit of Section 84 IPC.

83. Assertion: Any person, who is suffering from any kind of mental illness is called “medical insanity,” however
“legal insanity” means, person suffering from mental illness should also have a loss of reasoning power.
Reason: Every person who is mentally ill is not ipso facto exempted from criminal responsibility.
(a) Both A and R are true and R is correct explanation of A.
(b) Both A and R are true but R is not correct explanation of A
(c) A is true but R is false.
(d) A is false but R is true

Head Office: 127, Zone II, MP Nagar, Bhopal |+91-7676564400| https://www.toprankers.com Page 21 of 40
84. Despite having a medical history of insanity proven by evidence in court, the court convicted the accused based
on his subsequent conduct, namely, concealing the weapon, bolting the door to avoid arrest, and absconding
thereafter as the said acts. Is the ruling of the court valid?
(a) Yes as the subsequent acts of the accused will be held by the court to be a display of consciousness of the
guilt.
(b) No, as once the accused discharges its burden of proving that the said act is done while under a state of
insanity, the court cannot make an order otherwise against the accused.
(c) Yes, as the subsequent acts of accused proves that the accused was capable of knowing the nature of the acts
and know that such act is contrary to law.
(d) No, even though the subsequent acts of the accused states otherwise, it is important to note that his medical
history of insanity is proven by evidence in the court.

85. Soma suffered from somnambulism. Somnambulism is an unconscious state that is also known as sleep walking.
Her parents usually lock her door while she sleeps because previous incidents reveal that her sleep walking
almost led to the reason for her death because she was caught walking on a busy highway adjacent to their home
while she was sleeping waking. When Soma's parents left town for one day, they failed to inform the caretaker
that Soma's room should be locked or that she should keep a close eye on her while she sleeps. Soma left the
room and began walking towards the kitchen, where she killed the caretaker. Will she be held accountable?
(a) No, as per the principles of criminal jurisprudence an act does not constitute guilt unless done with a guilty
intention.
(b) Yes, soma will be held liable as an act does constitute guilt unless not done with a guilty intention
(c) Yes, as per the principles of criminal jurisprudence an act does constitute guilt unless done with a guilty
intention.
(d) No, soma will not be held liable as she did not have any intention to kill the caretaker.

86. Continuing with the previous example, can soma use insanity as a defense in this case?
(a) Yes, as all ingredients of section 84 are satisfied.
(b) No, as none of the essentials of section 84 have been satisfied.
(c) No, as none of the essentials of section 84 has been satisfied but can claim defense of Actus nonfacit reum
nisi mens sit rea.
(d) Yes, as a person with mental illness has no free will.

Passage (Q.87- Q.91): Section 27 of the Contract Act, 18721provides that: An agreement restraining a person
from carrying on a lawful profession, trade or business is void to that extent. However, an agreement not to carry
on within specified local limits, a business similar to the business of which goodwill is sold, can be enforced,
provided the limits of restraint are reasonable.

The section is general in nature, and declares all agreements in restraint of trade void, except in the case specified
in the exception. The doctrine of restraint of trade does not apply during the continuance of the contract for
employment and it applies only when the contract comes to an end. Section 27 of the Contract Act makes void
i.e. unenforceable, every agreement by which anyone is restrained from exercising a lawful profession, trade or
business of any kind.

Section 27, on promulgation of the Constitution of India, conferring the right to practice any profession or to
carry on any occupation, trade or business, the status of a fundamental right, under Article 19(1)(g) thereof, today
has a different connotation. Article 19(6) only clarifies that nothing contained in clause (g) shall affect the
operation of any existing law or prevent the State from making any law, imposing in the interest of general
public, reasonable restrictions on the exercise of right conferred by the said clause. Thus, restrictions, in the
interest of general public and if reasonable, to the fundamental right to practice any profession or to carry on any
occupation, trade or business, can be imposed only by law. The law of tort of unreasonable interference in

Head Office: 127, Zone II, MP Nagar, Bhopal |+91-7676564400| https://www.toprankers.com Page 22 of 40
carrying on business, in view of Section 27 of the Contract Act in force since 1872, was not the existing law
within the meaning of Article 19(6) of the Constitution.

87. A, B, C, D and E operated a Chartered Accountancy Firm. Their business was going on very well and they had
a long client roster. However, over time the five partners started having petty differences and decided to go their
separate ways. Since A was the original proprietor of the firm, B, C, D and E entered into an agreement with A
under which all of them or any of them, had undertaken not to carry on or be involved in any capacity in any
business competing with the business of A, even after leaving employment/association with A. A few years later,
B opened a rival Chartered Accountancy Firm. A seeks to prohibit B. Can he do so?
(a) Yes, because the agreement between A,B,C,D and E is void and unenforceable.
(b) Yes, the agreement between A,B,C,D and E is valid and enforceable.
(c) No, because the agreement between A,B,C,D and E is void and unenforceable .
(d) No, the agreement between A,B,C,D and E is valid and enforceable.

88. Anjana was working in the Electricity Department as a software engineer. Anjana left her job at the Electricity
Department to join a Multinational Corporation (MNC) in the same role. When she handed in her resignation,
she was told that she could not take up the job at the MNC. The reason cited by the department was that as per
the terms of the government agreement mandated by the State of Madhya Pradesh, she was not allowed to
compete by offering similar services to another company for five (05) years after termination of service. Is the
contention of the Electricity Department valid?
(a) No, because reasonable restrictions on competing are allowed
(b) Yes, because reasonable restrictions on competing are allowed
(c) No, because an agreement restraining a person from carrying on a lawful profession, trade or business is void
to that extent
(d) Yes, because the restriction was a fiction created by the State of Madhya Pradesh

89. A and B were two brothers who ran a shop selling fried flour bread with spicy chickpeas i.e., 'Chole Bhatoore'
in Delhi's Connaught Place. The name of the shop was Chacha de Chole Bhatoore. The food was so famous that
people used to come from far and wide to try the Chole Bhatoore. However, over time a ride arose between the
two brothers and they decided to part ways. B left the shop and started a new shop selling the same item right in
front of A's shop, by the name of 'Kaka de Chole Bhatoore'. As soon as B opened his shop, A started experiencing
a downfall in his customers and lost a lot of business. A suggested that B close his shop in exchange for Rs. 1
crore. B agreed and closed his shop. However, A failed to pay the amount to B. B sued A for breach of contract.
Will he succeed?
(a) Yes, because A and B entered into a valid agreement as per Section 27
(b) Yes, because A and B entered into a valid agreement as per Article 19
(c) No, because A and B did not enter into a valid agreement as per Section 27
(d) No, because A and B did not enter into a valid agreement as per Article 19

90. Dr. Ghanshyam was a physician who worked at Grey Sloan Memorial Hospital. He employed Freud as is
assistant for three years. As Dr. Ghanshyam’s assistant, Freud used to not only manage his appointments,
schedule his surgeries in the calendar, maintain his records etc., but also assist him during surgeries, take patient
histories etc. Freud was himself a medical graduate and wanted to use this experience to get practical insight into
the career of a physician. As part of his employment contract, Freud agreed not practice medicine anywhere else
in the world for the duration of three years. Is the contract between Dr. Ghanshyam and Freud valid?
(a) Yes, because it is an agreement in restraint of trade as per Section 27 of the Contract Act
(b) No, because it is not an agreement in restraint of trade as per Section 27 of the Contract Act
(c) Yes, because the agreement is independent of the application of the doctrine of restraint of trade
(d) No, because the agreement is not independent of the application of the doctrine of restraint of trade

Head Office: 127, Zone II, MP Nagar, Bhopal |+91-7676564400| https://www.toprankers.com Page 23 of 40
91. Popatlal was a lawyer. His family had been practice the profession for over 60 years. He had inherited his practice
from his father Hiralal, who himself had inherited the practice from his father, Dhanlal. However, Popatlal did
not enjoy being a lawyer and wanted to become a rapper. He decided to sell his legal practice, ‘Lal Sons and
Associates’ to Z along with the goodwill that had been commanded over 60 years of hard work. Popatlal agreed
to never practice as a lawyer in the state for the next 20 years as one of the terms of the contract with Z. Is the
contract between Popatlal and Z valid?
(a) Yes, because it is an agreement not to carry on within specified local limits, a business similar to the business
of which goodwill is sold
(b) No, because it is not an agreement not to carry on within specified local limits, a business similar to the
business of which goodwill is sold.
(c) Yes, because it is an agreement not to carry on within specified local limits, a business similar to the business
of which goodwill is sold with reasonable limits
(d) No, because it is an agreement not to carry on within specified local limits, a business similar to the business
of which goodwill is sold with unreasonable limits

Passage (Q.92- Q.96): In the interest of safe-guarding the health of a 10-year-old rape survivor, the Supreme
Courton Thursday allowed medical termination of her 30-week-old pregnancy even though the termination
period under Medical Termination of Pregnancy Act, 1971 is 24 weeks.
Justice PV Kunhikrishnan noted that even though there is an 80% chance of the baby surviving despite the
procedure, there are chances of severe complications impacting the 10-year-old's health.

"Since the victim child is only aged ten years, there is a chance for medical complication to her health.
Considering the entire facts and circumstances of the case, according to me, this is a case in which this Court
should invoke the jurisdiction keeping in mind the Almighty," the Court said in its judgement.

The judgment was passed on a petition moved by the 10-year-old survivor's mother seeking permission to
undergo medical termination of her pregnancy under the provision of the Medical Termination of Pregnancy
Act, 1971 and direction to the concerned hospital to conduct such termination procedure in accordance with the
law.

Upon considering the report and the age and other circumstances of the minor rape survivor, the Court decided
to permit the hospital to do the needful in accordance to the Medical Termination of Pregnancy Act, within a
week's time.

Further, the Court directed that if the baby is born alive, and the parents of the child are not willing to or not in
a position to assume the responsibility of the child, then the State and its agencies will have to assume full
responsibility of the child and medical support and other facilities as may be reasonably feasible, adhering always
to the principle of best interests of such child as well as the statutory provisions in the Juvenile Justice Act.

92. In light of the passage, which of the following options aptly summarise the approach of the Hon’ble Supreme
Court in matters pertaining to Medical Termination of Pregnancy Act?
(a) The Supreme Court shall decide the Medical Termination of Pregnancy solely on the ground of health of the
expecting mother.
(b) The Supreme Court shall decide the Medical Termination of Pregnancy only in rape cases.
(c) The Supreme Court shall decide the Medical Termination of pregnancy only in case of minor girls.
(d) The Supreme Court shall decide the medical termination of pregnancy predominantly on the ground of health
of the expecting mother.

Head Office: 127, Zone II, MP Nagar, Bhopal |+91-7676564400| https://www.toprankers.com Page 24 of 40
93. Sarvesh and Nidhi are a happily married couple and have planned parenthood. Subsequently, Nidhi gets pregnant
and is 27 weeks pregnant when she decides to terminate her pregnancy. However, the Doctor refuses to perform
abortion. She approaches the Court contending that she wants to terminate her pregnancy because she is not
ready to be a mother yet. The Medical reports show that if pregnancy is terminated at this stage, it might give
birth to a child with imperfect limbs. Based on the passage above, will Court allow abortion?
(a) The Court will allow termination as the choice of the mother is paramount in deciding termination of
pregnancy.
(b) The Court will allow termination as a woman who is not ready to be a mother means serious consequences
on the health of a mother.
(c) The Court will not allow termination as the Court will only consider the abnormalities in the child to be born
as a result of such termination.
(d) The Court will not allow termination as the totality of facts does not seem enough to justify termination at
such an advanced stage of pregnancy.

94. On the basis of the passage above herein, decide which of the following grounds for termination of pregnancy
seems to be the least probable one when court is deciding if they should allow termination of pregnancy?
(a) The married-unmarried status of the mother
(b) The expected abnormalities or birth-defects of an unborn baby
(c) The medical reports of the doctor who refused to allow termination of pregnancy.
(d) The level of awareness in a society towards abortion/medical termination of pregnancy.

95. Sumit (34-year-old) and Kamli (18-year-old) are unmarried couple where Sumit is already married to Rekha and
therefore, having extra-marital affair with Kamli. However, Kamli ends up getting pregnant with Sumit and her
baby. Kamli wants to keep the baby, even though she is being pressurised by Sumit to go for an abortion.
Reluctantly she decides to abort the baby. However, the matter reaches court as doctor refuses to perform the
surgery. The Supreme Court allows abortion on grounds of extra-marital pregnancy, despite being aware that
Kamli is reluctant for abortion. In light of the passage, was Court correct in its decision?
(a) The Court was correct as the marital status of a mother is one of the dominant factors to allow termination
of pregnancy.
(b) The Court was not correct as the marital status of a mother cannot be the sole factor to refuse termination of
pregnancy.
(c) The Court was not correct as choice of the expecting mother is also an important factor to be considered in
totality of facts and circumstances.
(d) Both (b) and (c).

96. Based on the understanding of the passage, which of the following does not seem to be an objective of Medical
Termination of Pregnancy Act?
(a) To ensure well-being of the unborn baby and mother.
(b) To ensure that women do not have discretion to decide termination of the pregnancy, without permission of
the Court.
(c) To ensure that the termination is not unregulated and for reasons which are not in the interest of justice.
(d) To ensure that there is a balance between the choice of expecting mother and other considerations like health
of unborn baby, expectations of the father etc.

Head Office: 127, Zone II, MP Nagar, Bhopal |+91-7676564400| https://www.toprankers.com Page 25 of 40
Passage (Q.97- Q.101): The Supreme Court on Wednesday held that in cases of execution of a will, there is no
place for the Courts to see whether the distribution made by the testator was fair and equitable to all of his
children. A bench of Justices Hemant Gupta and V Ramasubramanian held that the principles governing
Article 14 of the Constitution of India does not apply to dispositions under a will and the testators are not required
to make fair and equitable distribution of their properties to all of their children.
"In the matter of appreciating the genuineness of execution of a will, there is no place for the Court to see
whether the distribution made by the testator was fair and equitable to all of his children. The Court does not
apply Article 14 to dispositions under a will," the Court ruled

The exclusion of one of the natural heirs from the bequest, cannot by itself be a ground to hold that there are
suspicious circumstances, the Bench added.

By way of background, a couple, Mannar Reddiar and Adhilakshmiammal, had two sons VM Chandrasekaran
and VM Sivakumar and a daughter by name Kalavathy. Both the mother and father in their wills bequeathed
their properties to their sons and not the daughter in the ground that she had been provided sufficiently during
the time of her wedding.

The Supreme Court held that none of the circumstances which the High Court set out to rule in favour of
respondents created any suspicion regarding the execution of the wills.

"The law relating to suspicious circumstances surrounding the execution of a Will is already well-settled and it
needs no reiteration...Cases in which a suspicion is created are essentially those where either the signature of
the testator is disputed or the mental capacity of the testator is questioned," the Court said.

97. Anil and Tina are a married couple and have two children- Sonu and Neha. Though they loved their kids equally,
they did not treat Neha as equal to Sonu due to their conservative thoughts about women. As a result of their
conservative thought, they transferred all their property to Sonu by way of a will. Neha challenged this before
the Court, seeking equality in division of property. Decide
(a) The will cannot be challenged as the testators are not required to fairly and equally distribute the properties
amongst their successors.
(b) The will can be challenged as the distribution was not on just grounds.
(c) The will can be challenged as, even though the testators are not required to fairly and equally distribute the
properties amongst their successors, the distribution has to be on just grounds
(d) The will cannot be challenged has the parents have sole right to distribute their properties amongst their
children.

98. Hitesh and Sarla are siblings and live with their mother. At the time of their father’s death, he had transferred all
the property by way of will to his wife. However, after a few years, the wife also died and left behind the property
for her kids. Hitesh got the property registered in his name. Sarla challenged his inheritance and claimed an equal
share in the property. Decide.
(a) She can challenge the inheritance by her brother, because the will had only transferred the property to the
mother and not to the children.
(b) She cannot challenge the inheritance by her brother because the court cannot interfere even when there is an
unfair or unequal distribution of property for the successors
(c) This question can’t be answered because the facts are not complete.
(d) Cannot be determined

Head Office: 127, Zone II, MP Nagar, Bhopal |+91-7676564400| https://www.toprankers.com Page 26 of 40
99. Sridevi and Jahnvi are two sisters living with their grandmother who is extremely old and is suffering from age-
related blindness, memory loss and hearing loss. Sridevi used to take utmost care of her grandmother, whereas
Jahnvi used to avoid taking her care. Ultimately, just before their grandmother passed away, she transferred all
the property to Sridevi through a will. Jahnvi challenged this will on grounds of suspicion. Decide.
(a) The will can be challenged on grounds of mental incapability of the testator as grandmother was old and is
suffering from age-related blindness, memory loss and hearing loss.
(b) The will cannot be challenged on grounds of suspicion as there is no requirement to fairly and equally
distribute the properties amongst the successors.
(c) The will can be challenged on grounds of suspicion, since the grandmother was old and suffering from many
ailments.
(d) The will cannot be challenged on grounds of suspicion as it was Sridevi who used to take utmost care of her
grandmother and hence the grandmother transferred all the property to Sridevi.

100. Ritika and Mitika are two twin sisters, however, they did not get along well. Through a will, their father has
given one house each to both his daughters. His well read as follows:
“By way of this will, I bequeath all my properties to my daughters Ritika and Mitika, in equal proportions. The
house situated at Delhi shall belong to Ritika, whereas the house situated at Shimla shall belong to Mitika.”
However, Mitika challenges the will on the ground that since the value of house situated at Delhi is more than
the value of house situated at Shimla, it is not an equal distribution as per will. Decide.
(a) The will cannot be challenged as the court cannot interfere even in cases of unfair and unequal distribution
of properties by a testator.
(b) The will cannot be challenged even though it provided that the properties are divided in equal proportions,
it was up to the father to decide which property shall belong to whom.
(c) The will can be challenged because it provided that the properties are to be divided equally; however, the
properties were of unequal value, and hence the final distribution was not equal.
(d) They will can be challenged on grounds of unfair and unequal distribution, since the will itself provided for
distribution in equal proportions.

101. Ira is a widow and an illiterate woman and has two sons- Suresh and Naresh. She has two properties- one shop
and a house. Her son, Suresh, used to take care of the shop and Naresh was still studying in college. Just before
her death, she transferred all her property to Suresh via a will which wasn’t signed but was in her handwriting.
Can Naresh challenge this will?
(a) The will can be challenged on ground of suspicion, since the will was unsigned.
(b) The will cannot be challenged on ground of suspicion even though it was unsigned, because it was in the
testator’s handwriting.
(c) The will can be challenged because not only there was suspicion, there was also unfair and unequal
distribution of properties.
(d) The will cannot be challenged because even though it was unsigned, the court cannot interfere in unequal
and unfair distribution of properties.

Passage (Q.102- Q.105): Under the Consumer Protection Act, 2019, a complaint can be filed when a consumer
detects deficient in a service. However, the threshold of deficiency must fall under the ambit of the definition of
deficiency given under the said act. The act not only recognizes physical relationship of buyer-seller but also has
acknowledged services pertaining to e-commerce platforms. The burden of proving deficiency in service lies on
the shoulders of the person who attributes it. The Act imposes strict liability on the manufacturer for causing
harm by its defective products. So this means that the consumers can file suit against the manufacturer without
proving that the manufacturer was negligent. The mere necessity is to prove the defect in the product and the
damage or injury was caused by the product or service only.
Section 2 (11) of Consumer Protection Act, 2019 defines Deficiency of Service as “any fault, imperfection,
shortcoming or inadequacy in the quality, nature and manner of performance which is required to be maintained
by or under any law for the time being in force or has been undertaken to be performed by a person in pursuance

Head Office: 127, Zone II, MP Nagar, Bhopal |+91-7676564400| https://www.toprankers.com Page 27 of 40
of a contract or otherwise in relation to any service and includes (a) any act of negligence or omission or
commission by such person to the consumer and (b) deliberate withholding of relevant information by such
person to the consumer.”

102. Yogya was a well-known internet service provider in Mumbai's Thane area. Charu recently relocated to Mumbai
to study psychology at St. Xaviers College. Charu contacts Yogya to install a WiFi connection at her residence.
However, due to the ongoing bad weather conditions in Mumbai and the subsequent curfew, Yogya was unable
to provide the same on time leading to charu not being able to submit online exam form on time and was declared
failed in its first semester. Decide.
(a) Yogya will not be held liable for the poor service because the relevant conditions were not favorable to him.
(b) Yogya will be held strictly liable for the deficient service that caused harm to a cahru.
(c) Yogya will be held liable for insufficient service but will not face strict liability for it.
(d) Yogya will be held liable for deficient service because he was negligent in its actions.

103. The complainant (passenger) booked a return flight ticket with the Indijo
Airlines dated 15.02.2020 from Kochi to Bangalore and Bangalore to New Delhi and paid Rs.8,098/-. On
12.02.2020 while being at Kochi, the complainant came to know from other person that flight from Bangalore
to Delhi had been cancelled. The complainant contacted Airline and cancellation of flight was confirmed but no
reason of cancellation was stated. Due to the cancellation of flight, the complainant had to face lot of
inconvenience in making alternate arrangements for reaching Delhi and missed some business opportunities
resulting loss to the complainant. Decide
(a) There is no deficiency in service as there was no deliberate withholding of relevant information by the
airlines.
(b) There is a service deficiency on the part of Indijo Airlines because they were both negligent and omitted to
the consumer.
(c) The airlines will be held liable for Deficiency of Service since they failed to exercise their duty of informing
the passengers, resulting in a Deficiency of Service.
(d) In the absence of any evidences, it would not be possible to adjudge as to whether the passenger actually
faced the monetary loss or faced other difficulties.

104. Suppose, the connecting flight from Bangalore to Delhi was cancelled by the Indigo Airlines, however, there is
evidence that SMS to this effect was sent to the passenger intimating the cancellation of flight. However it was
the passenger that did not check his SMS on time. Will the instance still fall under the ambit of “deficiency in
service”?
(a) The airlines will still be held responsible as it failed to provide the service to its customer.
(b) The passenger will be deemed to be a customer of the airlines and thus it imposes strict liability on the service
provider.
(c) The airlines will not be held liable for deficiency in service as there was no omission or negligence on part
of the airlines in the present case.
(d) The airlines will be held liable as the passenger had to face lot of inconvenience in making alternate
arrangements for reaching Delhi and missed some business opportunities resulting loss to the complainant.

Head Office: 127, Zone II, MP Nagar, Bhopal |+91-7676564400| https://www.toprankers.com Page 28 of 40
105. Naman was a big foodie and run a food vlog channel on Youtube by the name of “Bhukkad_belly”. He is best
known for his explorations and reviews of street food in India’s various cities. Naman gives his audience a virtual
tour of street food and describes the best places to eat in India in this “Bhukkad_belly” Food Channel, which is
based on a Food Guide. Naman also has been featured on BBC News and Josh Talk. In times of covid, Naman
decided to go and help small businesses grow by giving several shoutouts to various street food vendors. One of
them was "baba ka dhaba," where Naman tried the meal and gave thumbs up to all his audience for this palace
to try "chole bhature." After this vlog went viral and "baba ka dhaba" gained instant popularity, the street food
outlet had lines of about 100 people the next day to try its famous chole bhature. A television news channel "kal
Tak" covered the incident and asked some customers for honest feedback on the food. Some customers were
pleased and satisfied with the food, but others criticised it for poor hygiene and the use of local oils in the
preparation of the food. Customers who were dissatisfied complained of deficiency in services. Decide
(a) Naman, including the food outlet, would bear accountability because he was the one who suggested to its
audience that they visit "baba ka dhaba."
(b) "Baba ka dhaba" will be held liable for poor service since consumers were dissatisfied with the taste of the
food served.
(c) "Baba ka dhaba" will not be held liable for poor service because consumers were pleased and satisfied with
the meal served.
(d) The current incidence will be classified as a deficiency in service.

Head Office: 127, Zone II, MP Nagar, Bhopal |+91-7676564400| https://www.toprankers.com Page 29 of 40
SECTION - D: LOGICAL REASONING

Directions (Q.106-Q.135): Read the passage given below and answer the questions that follow-

Passage (Q.106-Q.110): The conviction of Aung San Suu Kyi, Myanmar’s deposed State Counsellor and pro-
democracy leader, in a corruption case, shows nothing but the desperation of the junta to silence her. For the
military, which seems determined to destroy the Southeast Asian nation’s popular democracy, Ms. Suu Kyi,
under house arrest ever since the February 2021 coup, remains the enemy number one. The conviction, in a
kangaroo court, was based on the testimony of the former Chief Minister of Yangon who claimed that he had
handed over to her $6,00,000 and gold in return for favours. The prosecution has presented no evidence. She
was convicted earlier on five other charges and sentenced to six years in jail. In the corruption case, the court
jailed her for five years. The junta has slapped more cases on her, with the clear objective of keeping the 76-
year-old leader in prison. Since the coup, it has arrested 10,300 political prisoners, including most of the
elected lawmakers of Ms. Suu Kyi’s National League for Democracy. The forces have also killed at least
1,798 civilians and threatened to “annihilate” all opponents.

Myanmar’s military is one of the most stubborn enemies of democracy and basic human freedoms, having ruled
for nearly 50 years using brute force. But even in the darkest moments of Myanmar’s past, there was popular
resistance. And over the past three decades, Ms. Kyi has been the embodiment of that resistance. Between 1989
and 2010, she spent 15 years under house arrest. The military, faced with international isolation and growing
domestic anger, agreed to release her and share power with civilians through a quasi-democratic arrangement.
They barred her from becoming President and reserved key portfolios, including the Defence Ministry, for the
Generals. Still, the 2015 and 2020 elections saw overwhelming public support for her party, and the country
witnessed, barring the military crackdown on Rohingya Muslims, relative stability and growing economic
opportunities. But the military was worried whether an increasingly popular and powerful Ms. Suu Kyi, after her
second consecutive election, would clip its privileges. It was this fear that prompted the Generals to stage another
coup. They may have succeeded in reversing Myanmar’s limited experiment with democracy, but the coup has
also wreaked havoc on the country. The Opposition has taken up arms, pushing the country to the brink of civil
war. A nationwide strike has crippled the country’s economy. The political opponents of the coup have also
formed an alternative unity government. So far, the military has managed to cling on to power through sheer
repression. But it is not a sustainable model. Even silencing Ms. Suu Kyi would not help the junta tighten its
control over a divided, impoverished, and rebellious country.

106. What can be best represented as the central idea of the passage?
(a) Myanmar has a bleak political future, with its leader and citizenry handicapped of their rights.
(b) Democracy lies in the heart of a country’s development, disrupting which can wreak havoc.
(c) A country is defined by the success of its leader.
(d) The democratic procedure, if duly followed, leads to the country’s flourishment.

107. Which of the following can be inferred from the above passage?
(a) Myanmar as a country has always championed democracy.
(b) Suu Kyi is well known for her advocacy of non-violence.
(c) Courts, after much deliberations, held Ms. Suu Kyi of several charges.
(d) The military, since its inception, has refused to devolve powers to democratic institutions.

108. Which of the following can be assumed from the above passage?
(a) There is no likelihood of restoration of democracy in Myanmar.
(b) NLD has worked for the betterment of the citizenry and has upheld its aspirations.
(c) Suu Kyi is the most daunting political figure across the country.
(d) A nationwide strike has crippled Myanmar's economy.

Head Office: 127, Zone II, MP Nagar, Bhopal |+91-7676564400| https://www.toprankers.com Page 30 of 40
109. What purpose does the boldfaced statement serve in the passage?
(a) It is an argument to support that a military takeover has brutal repercussions.
(b) It is a premise to support that a military takeover has brutal repercussions.
(c) It is a premise supporting the early release of people arrested on false accusations.
(d) It is an argument supporting the early release of people arrested on false accusations.

110. Which of the following, if true, weakens the author’s argument?


(a) Suu Kyi’s short government tenure provided a much-needed stability and economic growth after a long time.
(b) The Junta fears the growing popularity of Aung San Suu Kyi among the people of Myanmar.
(c) The crisis afflicting Myanmar fins no respite after the coup by the junta.
(d) Aung San Suu Kyi’s party won by a very small margin in the 2020 elections.

Passage (Q.111-Q.115): In history, fiction and historical fiction, great leaders have risen to the pulpit and the
occasion. Mark Antony, in Shakespeare’s Julius Caesar, asks “friends, Romans and countrymen”, “What cause
withholds you then, to mourn for him?”, after Caesar’s assassination. Closer home, in the hallowed halls of
India’s Parliament, Jawaharlal Nehru made a newborn country proud of its tryst with destiny, poverty and
Partition notwithstanding. In 2022, though, the era of the inspiring speech seems long over. As minister after
minister deserted the embattled prime minister of Britain last week. Leader of Opposition Keir Starmer quipped
in the House of Commons that this is “the first case of sinking ships fleeing the rat”. With a single turn of phrase,
he managed to call Prime Minister Boris Johnson a pest, while implying that the top leadership of the
Conservative Party, including those who are now frontrunners as Johnson’s successors are disloyal and
themselves in decline. In an age of one-upmanship and cutting remarks in 240 characters, Starmer’s statement
stands out for its brevity and wit. But that’s all. The perfect punchline pointed to the missing story. Perhaps, in
an earlier time, before everyone had a voice and each one had a lot more to say, Britain’s Leader of Opposition
might have made a speech that befits the crisis — an erosion of political trust in an economy still reeling from a
once-in-a-century pandemic — one that inspired even as it derided. But in a world of diminishing attention spans
and polarised politics, a smart one-liner, it seems, is the best that the politician can summon, on either side of the
aisle.

111. What is the central idea in the passage as conveyed by the author?
(a) A clever one-liner seems to be the best a politician can muster in the present world.
(b) Starmer’s statements on Boris Johnson reveal an irony in itself.
(c) The era of inspiring speeches such as that of Mark Anthony and Nehru has disappeared.
(d) Starmer’s comments in the prevalent times seem nothing less than a stab in the back.
112. According to the given passage, which among the following is true?
(a) Contemporary leaders have performed better than usual in response to a special situation or event.
(b) Mark Anthony’s example is that of inspiring speech from leaders from the past.
(c) Keir Starmer’s speech, though witty, lacked a rationale.
(d) The length of the address of Keir Starmer was adequate in the given scenario.
113. What does Keir Starmer signify by - “the first case of sinking ships fleeing the rat”?
(a) When people abandon an enterprise when it is least expected.
(b) When people abandon an enterprise when it is about to excel.
(c) When an enterprise collapses on witnessing its people deserting it.
(d) When people abandon an enterprise when it is in inception.
114. Which among the following can be inferred from the given passage?
(a) Starmer’s speech would be welcomed by most of the Britishers.
(b) The economy of Britain is still some distance away from normalcy.
(c) Diminished attention spans in people resulted in Starmer’s speech.
(d) Britain is known for its war of wits in politics.

Head Office: 127, Zone II, MP Nagar, Bhopal |+91-7676564400| https://www.toprankers.com Page 31 of 40
115. As per the above passage, which of the following is correct about the world?
(a) Keir Starmer’s speech was enveloped in a paradox.
(b) The attention span of people is irrelevant to the influx of news.
(c) Today’s politicians lack the political depth needed to run the country.
(d) Inspiring speeches are no longer sought after.

Passage (Q.116-Q.120): GoI must follow through on the Supreme Court’s suggestion that India needs a
comprehensive bail legislation, on the lines of the UK Bail Act. Around 70% of the prison population are
undertrials, most of them poor and therefore unable to secure monetary bail. SC took note of CrPC’s colonial
biases that continue – police are quick to arrest citizens, and this puts poor and/or poorly educated citizens at a
huge disadvantage as they are unable to take advantage of the legal system, unlike those with education and
affluence and/or influence. A law collating the laws in one place can check arbitrariness. But a problem, which
SC notes too, is the attitude of judicial officers. SC wondered whether the low rate of conviction tends to make
judges adopt a negative attitude towards bail. Instead of a punitive approach to bail, SC has now directed that
bail applications should be disposed of within two weeks in the normal course. This is a reasonable diktat.
Sometimes, bail applications are kept pending for months. Unnecessary arrest coupled with no bail is a double
whammy. SC has reiterated that arrest is a draconian measure to be used sparingly, otherwise the “impression
that it is a police state” will gain ground, which would be a disturbing description of a democracy. SC’s
scepticism over the compliance with its 2014 Arnesh Kumar judgment reveals how well-intentioned verdicts are
being subverted. The judgment had directed cops to diligently record specific reasons necessitating arrest, and
magistrates to grant bail if the reasons don’t make a persuasive case. Disciplinary action against errant officers
may produce minor changes. But quick bail is a good corrective against unnecessary arrests.

116. “Courts tend to think that the possibility of a conviction being nearer to rarity, bail applications will have to be
decided strictly, contrary to legal principles.” What role does the above statement play in the passage?
(a) This strengthens the arguments presented in the passage.
(b) This weakens the arguments presented in the passage.
(c) This statement is a hypothesis of the author’s arguments.
(d) This statement is irrelevant to the information of the passage.

117. Out of the following, what must be assumed for the author’s arguments to hold?
(a) Quick bails need to be given in cases where the charges hold no water.
(b) Provisions related to bail applications are easy to find and apply.
(c) Bail applications for the poor are never approved in the Indian judicial system.
(d) The Supreme Court’s decision will check political arbitrariness.

118. What is the central idea of the passage, as conveyed by the author?
(a) Most prisoners in Indian jails have had their bail applications rejected.
(b) In India, it is an arduous task for the poor to get their bail applications approved.
(c) The government should take up the Supreme Court’s idea on the bail legislation.
(d) Disciplinary action against the errant officers would help in getting quicker bails.

119. What does the author convey by ‘double whammy’?


(a) Not granting bail, on top of unnecessary arrests, is a cherry on the top.
(b) Denying bails even after unnecessary arrests is a doubly problematic.
(c) The judicial system needs to either approve bails or prevent unnecessary arrests.
(d) If quick bail is given, unnecessary arrests may be made more often.

Head Office: 127, Zone II, MP Nagar, Bhopal |+91-7676564400| https://www.toprankers.com Page 32 of 40
120. According to the author, which of the following would be a logical course of action post the SC direction?
(a) The police become complacent and arrest only when it is a high crime.
(b) Disciplinary action against the errant officers is given utmost importance.
(c) The Government of India expedites the bails of the 70% of the prison population that are undertrials.
(d) The government assimilates and creates one comprehensive law on bails.

Passage (Q.121-Q.125): Efforts to curb greenhouse-gas emissions and the impacts of global warming will fall
significantly short without drastic changes in global land use, agriculture and human diets, leading researchers
warn in a high-level report commissioned by the United Nations. The special report on climate change and land
by the Intergovernmental Panel on Climate Change (IPCC) describe plant-based diets as a major opportunity for
mitigating and adapting to climate change ― and includes a policy recommendation to reduce meat consumption.
On 8 August, the IPCC released a summary of the report, which is designed to inform upcoming climate
negotiations amid the worsening global climate crisis. More than 100 experts, around half of whom hail from
developing countries, worked to compile the report in recent months. “We don’t want to tell people what to eat,”
says Hans-Otto Pörtner, an ecologist from IPCC. “But it would indeed be beneficial, for both climate and human
health, if people in many rich countries consumed less meat, and if politics would create appropriate incentives
to that effect.” Researchers also note the relevance of the report to tropical rainforests, with concerns mounting
about accelerating rates of deforestation. The Amazon rainforest is a huge carbon sink that acts to cool global
temperature, but rates of deforestation are rising, in part because of the policies and actions of the government
of Brazilian President Jair Bolsonaro. Unless stopped, deforestation could turn much of the remaining Amazon
forests into a degraded type of desert, and could release more than 50 billion tonnes of carbon into the atmosphere
in 30 to 50 years, says Carlos Nobre, a climate scientist at a University. “That’s very worrying for the earth,” he
rightly says.

121. Out of the following, what is the author's central theme in the above passage?
(a) Why is it so important to take action on curbing global warming?
(b) What comes in the way of rising greenhouse-gas emissions?
(c) What could hamper the efforts to curb greenhouse-gas emissions?
(d) What role could plant-based meat play in curbing global warming?

122. As per the passage, which of the following is a challenge?


I. Reducing the consumption of animal-based meat in developing countries.
II. Increasing the rates of deforestation in Latin American countries.
III. Decreasing the amount of carbon that could be released into the atmosphere.
(a) Only I (b) Only III (c) Both I and III (d) I, II & III

123. What is the difference between the statements of Carlos Nobre and Hans-Otto Pörtner?
(a) The statement of Carlos is a lot more authoritative than that of Pörtner.
(b) The message of Carlos is quite straightforward, whereas this can’t be said about Pörtner.
(c) The statement of Carlos is about policy implementation, while Pörtner acknowledges a policy
recommendation.
(d) Carlos warns people against something, whereas Pörtner appeals to the public.

124. What is the correct representation of the author’s opinion in the passage?
(a) Some marginal resource use changes would come in handy in complementing the efforts to curb greenhouse
gas emissions.
(b) Politics could play a significant role in appealing to people to consume less food.
(c) The Amazon rainforest needs to be preserved to avoid creating a huge global carbon soaring.
(d) Thirty to fifty years hence, most of the species, flora and fauna, will become extinct.

Head Office: 127, Zone II, MP Nagar, Bhopal |+91-7676564400| https://www.toprankers.com Page 33 of 40
125. If the information in the passage is correct, then which of the following is also be true except?
(a) The representatives of IPCC are pretty vocal about what they want from the people.
(b) The taste of a plant-based diet could be developed to be similar to an animal diet.
(c) The release of more than 50 billion tonnes of carbon would spell doom for us in 10 to 40 years.
(d) All of the above.

Passage (Q.126-Q.130): That policies change with governments may appear acceptable in a democracy, but no
court will allow a transformation so fundamental that rights and entitlements that accrued during an earlier
regime are abandoned or frustrated. In 2014, the Amaravati region was chosen as the site of the capital of Andhra
Pradesh, the residuary State left after the creation of Telangana, but work was stopped after the present YSRCP
regime took over. Instead, Chief Minister Y.S. Jagan Mohan Reddy mooted the idea of ‘decentralised’
development, by which he meant that the State will have Amaravati as the legislative capital, while
Visakhapatnam will be the executive capital, and Kurnool the seat of the High Court. In a stern rebuff to the
‘three capitals’ idea of the present regime, a Full Bench of the State’s High Court has ruled that it cannot abandon
the project to develop Amaravati as the capital city after over 33,000 acres had been given up by farmers and
₹15,000 crore sunk in it over development expenditure. Holding the State government to its promise of
developing the region into the capital city, it has directed the government to complete the required developmental
work in Amaravati within six months. As a consequential relief to the farmers who had given up their land for
the specific purpose, the court has asked the State and the Andhra Pradesh Capital Region Development
Authority to develop the surrendered land and deliver reconstituted plots to them within three months.

In political terms, the decision to locate the three organs in different cities was possibly motivated by a
wish to undo a key policy measure of the previous TDP regime, as well as negate speculative gains the
erstwhile rulers may have allegedly made by choosing Amaravati. However, it was projected as a measure
to decentralise governance and take the fruits of development to all parts of the State. After farmers approached
the court, the government sought to render the matter infructuous by repealing its decentralisation law, the one
that it enacted in 2020 for spreading the capital city and proposing ‘inclusive development’ of all regions.
However, the Bench took note of the government’s intention to pursue its multiple capital cities plan and decided
that it will adjudicate on the petitioners’ grievances. It held that the State legislature lacked the competency to
shift the organs of the State. The verdict, if undisturbed by the Supreme Court, may put an end to attempts to
shift the capital city out of Amaravati. A welcome feature of the verdict is that it has invoked the doctrines of
constitutional trust and promissory estoppel to prevent a regime from going back on its promises to citizens.

126. What best represents the central idea of the passage?


(a) A.P. High Court judgment on Amaravati protects governance from political whimsy.
(b) Trust and faith in courts guard tenets of federalism.
(c) India isn’t adapted to the three capital model to sustain.
(d) Decentralisation is a hallmark of democracy that cannot be compromised under any circumstances.

127. Which of the following, if true, can strengthen the arguments above?
(a) South Africa, a country with a three-capital model, sees progress in states.
(b) Mauritius is planning to convert back to a single capital system, after doldrums faced in its administration.
(c) High court interference in executing government policies can lead to interrupted and stalled growth of the
states.
(d) The Central Government welcomed the idea of ‘decentralised’ development and taking the fruits of
development to all parts of the State.
128. Which of the following forms the valid assumption of passage?
(a) Regimes frequently act according to their whims.
(b) YSRCP regime is ignorant of the plight of farmers.
(c) Andhra Pradesh has a history of continuous regime change.
(d) High court decisions are binding on the governments.
Head Office: 127, Zone II, MP Nagar, Bhopal |+91-7676564400| https://www.toprankers.com Page 34 of 40
129. Assuming the above arguments and facts to be true, which of the following is most likely to be agreed by the
author?
(a) Courts are unvigilant of executive actions being undertaken.
(b) The State’s High Court verdict preserves doctrines of constitutional trust.
(c) The administration is excused from the interference of the courts.
(d) State’s High Court decisions are cast in stone.

130. What purpose does the boldfaced statement serve in the passage?
(a) It is a premise which supports the central idea of the passage.
(b) It is one of the author's arguments, which has been supported above.
(c) It plays the role of counter premise in the passage.
(d) It is a premise used to negate the author’s argument.

Passage (Q.131-Q.133): Amid the general euphoria over crossing the 1 billion landmark of total vaccinations,
reports of detection of presumably the most contagious strain of the coronavirus, Delta Plus, have started
emerging in parts of the country. If Delta created havoc during the second wave, with the mutation first being
detected in Maharashtra and Punjab, cases of AY.4.2, called Delta Plus, have been found during genomic
sequencing in India. AY.4, another sublineage of Delta, has already been detected in genomic samples from
Madhya Pradesh and Maharashtra. So there is no room for complacency. It's in this context that reports of people
skipping or delaying their second shot of the vaccine is worrying. They ought to understand that despite being
fully vaccinated, Israel is witnessing breakthrough infections, forcing it to go for booster shots. Russia has
announced a week-long lockdown as cases continue to go north, partially because of extremely high vaccine
hesitancy.

One of the takeaways of the otherwise disastrous second wave was the ramping up of the healthcare
infrastructure. While the administration surely deserves credit for combating the scourge with its humongous
vaccine rollout, the civil society and judiciary did their part in prodding the government to do the right thing.
From forcing it to amend its vaccine purchase policy and streamlining the supply of scarce oxygen to offering
relief to the next of kin of Covid victims, they kept the government on its toes and strengthened democracy.
Their participatory role cannot be lost sight of.

131. Which of the following statement(s) cannot be inferred from the given facts of the passage?
(a) Delays in the second shot of vaccine by the people should be handled immediately by the authorities.
(b) Crossing the milestone of one Billion vaccinations should not make the authorities and the public complacent
about the pandemic.
(c) As soon as the nation crossed the mark of one Billion vaccinations, the flaws in the vaccines emerged.
(d) The judiciary acted as a watchdog and a guiding figure to the policymakers.

132. Which of the following arguments has been considered lenient in second shots of the vaccine?
(a) Delinquent attitude that the public is showing towards the second dose of the vaccine.
(b) Authorities' lagging while dealing with the vaccination process.
(c) Complacency attributed to the government's actions during the first phase of the Delta variant.
(d) Delayed actions that the government is showing in its improvement in the healthcare infrastructure.

133. Which amongst the following statement(s) would always be TRUE of the passage?
I. That crossing the 1 Billion mark of vaccination gives immunity to India to lower its Covid pandemic's
restrictions.
II. That limiting the administration's scope was a violation of the Constitutional principles.
(a) Only I (b) Only II (c) Both I and II. (d) Neither I nor II.

Head Office: 127, Zone II, MP Nagar, Bhopal |+91-7676564400| https://www.toprankers.com Page 35 of 40
134. In a park, chairs are arranged as follows. Chair C is 5 m to the West of chair B, which is 10 m to the South of
chair A. Chair C is 12 m to the North of chair D, which is 11 m to the South of chair F. Chair E is 20 m to the
East of chair A and 15 m to the North of chair G. If chair M is 20 m to the East of chair B, then how far and in
which direction is chair G with respect to chair M?
(a) 10 m, South (b) 5 m, North (c) 10 m, North (d) 5 m, South

135. Three girls A, B and C and four boys P, Q, R and S are sitting in a row such that no two girls sit next to each
other. Q is sitting at the extreme left. P is the only neighbour of A. Q and R are the only two boys sitting adjacent
to each other. Who is sitting in the middle?
(a) B (b) S (c) C (d) Cannot be determined

Head Office: 127, Zone II, MP Nagar, Bhopal |+91-7676564400| https://www.toprankers.com Page 36 of 40
SECTION - E: QUANTITATIVE TECHNIQUES

Passage (Q.136-Q.140): Kingfisher group of company acquired four team of IPL viz. Mumbai Rock, Delhi
Rock, Patna Rock and Pune Rock. They invest some money (expenses) and earned revenue by selling its tickets.
After completion of league the results (expenses, revenue, profit and loss) of the teams for the year 2015 were
summarized. Chairman of Kingfisher group found that the profit of Patna Rock and Pune Rock were same, the
revenue earned by sell of Patna Rock tickets were the same as those of Delhi Rock. Profits of Mumbai Rock
were 10.75% of revenue earned by its tickets sales, whereas the profits of Delhi Rock were 20% of revenue
earned by its tickets sales. While the total expenses of Patna Rock were 2.5 times its profits, revenue earned by
sells the ticket of Pune Rock were 1.5 times of its profits. The total expenses of Patna Rock were 12 million and
of Mumbai Rock were 10% less than those of Patna Rock. Profit is defined as the difference between sales and
total expenses.

136. Profit of Pune Rock was what percent of its sales?


(a) 66.67% (b) 61.67% (c) 59.67% (d) 52.67%

137. Profit of all four team together which was acquired by Kingfisher group was what percent of its sales?
(a) 28.96% (b) 26.96% (c) 22.87% (d) 21.81%
138. If all four teams Mumbai Rock, Delhi Rock, Patna Rock and Pune Rock donate 20%,40%,10% and 50% of its
profit respectively to a school of basic education for slum children, total donated money is what percentage the
profit of all companies together ?
(a) 35.41% (b) 37.41% (c) 31.41% (d) 39.41%
139. Which team has highest efficiency?
𝑝𝑟𝑜𝑓𝑖𝑡
Efficiency = × 100
𝑠𝑎𝑙𝑒
(a) Mumbai rock (b) Delhi rock (c) Patana rock (d) Pune rock

140. Find the ratio of average sells to average profit of all teams together ?
(a) 11.13 : 4.56 (b) 17.15 : 8.93 (c) 13.22 : 3.56 (d) 13.21 : 3.59
Passage (Q.141-Q.145): There are five different types of bats (i.e. A, B, C, D and E) are available in a store
with different quantity and at different price. Number of type ‘E’ bats in store is 80% of its per piece price and
price of per type ‘A’ bat is Rs. 75 more than that price of per type ‘E’ bats. Total number of type ‘C’ bats are 70
less than that of number of type ‘E’ bats. Total number of type ‘A’ and type ‘B’ bats are 15 less than that of total
number of type ‘E’ bats. The ratio of price of per piece of type ‘E’, type ‘D’ and type ‘B’ bats is 11 : 23 : 17.
Total price of per piece of all five types of bats is Rs. 2175 and total number of type ‘D’ bats are 400 less than
its price per piece bat. Ratio of total number of type ‘A’ & type ‘B’ bat is 16 : 25 and price of per piece of type
‘C’ bat is Rs. 25 less than that of price of per piece of type ‘D’ bat.
141. Price of per piece of type D bats is what percent more than price of per piece of type E bat?
1 1 1 1
(a) 101 11 % (b) 97 11 % (c) 95 11 % (d) 109 11 %

142. Total number of type A & D bats together is how much more than total number of type E bats?
(a) 35 (b) 25 (c) 28 (d) 24

143. Find the ratio of price of per piece of type A bat and to price of per piece of type B bat ?
(a) 14 : 19 (b) 14 : 17 (c) 14 : 23 (d) 14 : 13
144. If Ram want to buy three type of bats D, six type of bats C & two type of bats E from store, then find how much
amount should Ram have to spend?
(a) 5575 Rs. (b) 5618 Rs. (c) 5821 Rs. (d) 5733 Rs.

Head Office: 127, Zone II, MP Nagar, Bhopal |+91-7676564400| https://www.toprankers.com Page 37 of 40
145. Find average number of type ‘A’, ‘B’, ‘D’ & ‘E’ bats available in store?
(a) 150 (b) 120 (c) 160 (d) 165

Directions (Q.146-Q.150): Read the data carefully and answer the following questions.
Bar graph given below shows the number of northbound and southbound migrations of Gray whale over different
months of a year.

146. In which of the following month, difference between number of northbound migration of Gray whale and
southbound migration of Gray whale is the maximum and what is that difference?
(a) Jun, 40 (b) Aug, 50 (c) Apr, 60 (d) May, 30

147. For which pair of the following months, northbound migration of Gray whale count is lowest and southbound
migration of Gray whale count is highest?
(a) May - Oct (b) Jun - Sep (c) Feb - Jul (d) Jun - Aug

148. For which of the following months, total number of northbound and southbound migrations of Gray whale is
more than or equal to 70?
(a) May, Aug and Oct (b) Apr, Aug and Sep
(c) Jun, Aug and Dec (d) Feb, May and Jul

149. What is the difference between total number of northbound migrations of gray whale in the first six months and
total number of southbound migrations of gray whale in the last six months?
(a) 5 (b) 10 (c) 20 (d) 15

150. Total number of northbound and southbound migrations of gray whale in Oct is what percent of that in May?
(a) 90% (b) 75% (c) 80% (d) 60%

Head Office: 127, Zone II, MP Nagar, Bhopal |+91-7676564400| https://www.toprankers.com Page 38 of 40
Notes:-

Head Office: 127, Zone II, MP Nagar, Bhopal |+91-7676564400| https://www.toprankers.com Page 39 of 40
Notes:-

Head Office: 127, Zone II, MP Nagar, Bhopal |+91-7676564400| https://www.toprankers.com Page 40 of 40

You might also like